Сохранен 563
https://2ch.hk/sci/res/566163.html
24 декабря 2023 г. Архивач восстановлен после серьёзной аварии. К сожалению, значительная часть сохранённых изображений и видео была потеряна. Подробности случившегося. Мы призываем всех неравнодушных помочь нам с восстановлением утраченного контента!

Тред тупых вопросов

 Аноним 10/10/22 Пнд 07:05:21 #1 №566163 
videoplayback (26).mp4
Даем умные ответы на тупые вопросы

https://2ch.hk/sci/res/564437.html#566156
Аноним 10/10/22 Пнд 07:07:02 #2 №566165 
>>566163 (OP)
Как долго эта штука будет работать
Аноним 10/10/22 Пнд 09:20:24 #3 №566168 
Удваиваю вопрос >>566165
Алсо, видно же, что в максимальной точке шарик взлетает на высоту гораздо больше, чем та с которой он скатывался через дырку. Похоже на наебалово, возможно в нижней деревяшке стоит какая-нибудь хитрая схема с электромагнитом, который своим импульсом придает шарику дополнительную скорость. Иначе я х.з. как это согласовать с з.с.
Аноним 10/10/22 Пнд 12:38:46 #4 №566172 
В чем рахзница между псевдо- и квази-?
Аноним 10/10/22 Пнд 19:08:26 #5 №566178 
>>566172
>псевдо
Фейк, обманка.
>квази
Типа наподобие, похожее
Аноним 10/10/22 Пнд 19:58:25 #6 №566179 
>>566172
Псевдохуйня - нечто, при поверхностном рассмотрении кажущееся хуйней, но на деле ею не являющееся.
Квазихуйня - нечто, на самом деле не являющееся хуйней, но при определнных условиях её подобное.
Аноним 11/10/22 Втр 09:30:05 #7 №566193 
>>566165
Пока шарик не размагнитится.
Впрочем, только человечек с iq под 160 сможет благодаря этому эффекту создать ротор, вращающийся внутри статора.
мимо сделал это 12 лет назад
Аноним 11/10/22 Втр 12:04:12 #8 №566201 
>>566165
Там есть скрыты механизм с внешним питанием.
Аноним 11/10/22 Втр 13:30:52 #9 №566202 
>>566201
Человек-рентген в треде.
Всем облачиться в свинцовый скафандр.
Аноним 11/10/22 Втр 21:02:38 #10 №566207 
16654816011400.mp4
>>566163 (OP)
Аноним 11/10/22 Втр 22:07:01 #11 №566210 
Почему мы живём в говне, с нейросетью способной пролистывать действия за 5 минут до их свершения.
Аноним 11/10/22 Втр 22:09:16 #12 №566211 
>>566210
Почему эти нейросети не встраивают во все.
Аноним 11/10/22 Втр 22:37:36 #13 №566212 
1544914815676-0.jpg
>>566163 (OP)
Привет анончики.
Как можно заново использовать пластик в домашних условиях? Хочу переплавить и отлить в форму но ничего не выходит.
Что я делаю не так? Как пластик перелить в новую форму?
Покупной пластик не предлагать т.к. денех нет.
Аноним 11/10/22 Втр 22:45:18 #14 №566214 
>>566210
>Почему мы живём в говне
Потому что ты хуесос и терпилы, сосёшь хуи и жрёшь говно т.к. тебе ГЛАВНЫЙ приказал, а ты послушно исполняешь.
Аноним 11/10/22 Втр 22:55:33 #15 №566215 
1633950322757.jpg
>>566163 (OP)
Вот допустим я насрал и нассал в тарелку, и буду этим пытаться, делая так по кругу.
Аноним 11/10/22 Втр 23:46:55 #16 №566216 
>>566212
Гугли разницу между термореактивными и термопластичными полимерами.
Аноним 12/10/22 Срд 10:28:11 #17 №566224 
йуйцуйцуйцу.jpg
>>566163 (OP)
Могло бы появиться потомство у нашего древнего предка, типа австралопитека, и у современного человека?

Вроде гены должны совпадать по большей части, не?
Аноним 12/10/22 Срд 12:44:37 #18 №566225 
>>566224
Очень сомневаюсь. Точного ответа никто не знает, но анатомически австралопитеки и современные люди отличаются сильно. Самка австралопитека точно не смогла бы выносить гибрида.
Аноним 12/10/22 Срд 13:21:51 #19 №566226 
image.png
Почему одни животные просто и молча трахают друг друга, и идут по своим делам, забыв об этом. Другие идут на невероятные ухищрения и половой диморфизм, дабы спариться. Вот этот щегол танцует на полянке, которую он расчистил. А на ветке сидит невзрачная самка, причем эту ветку щегол чем-то начистил до блеска. Или вот птичка с бесполезной соплей висящей с носа, которая дико орет самке прямо в морду, чтоб произвести впечатление
https://www.youtube.com/watch?v=kI0K5j3qm2g
Аноним 12/10/22 Срд 14:20:42 #20 №566227 
>>566226
Потому что когда самкам дают выбор, а не просто насилуют, вид выраждается в какое-то ущербное говно и вымирает в итоге.
Как павлины например. Совершенно нежизнеспособное говно.
Аноним 12/10/22 Срд 14:23:11 #21 №566229 
>>566227
Инцел плез, павлины прекрасно живут в естественной среде
Аноним 12/10/22 Срд 15:51:14 #22 №566231 
>>566229
Не рвись куколд.
Лучше скажи как павлины выживают.
Аноним 12/10/22 Срд 15:55:36 #23 №566232 
image.png
>>566231
>ти по рвался
Аноним 12/10/22 Срд 17:27:23 #24 №566237 
>>566232
Зачем ты свою фотку прислал вместо ответа порванный куколд? Ручки трясутся, промахнулся?)
Аноним 12/10/22 Срд 21:12:06 #25 №566243 
>>566226
>Почему одни животные просто и молча трахают друг друга, и идут по своим делам, забыв об этом.
Не бывает такого.
>Другие идут на невероятные ухищрения и половой диморфизм, дабы спариться.
Половой отбор называется, порвак-младшекласник. Узнаешь в школе в 7 классе.
Аноним 12/10/22 Срд 21:25:21 #26 №566244 
>>566243
>Не бывает такого.
пруфы
>Половой отбор называется, порвак-младшекласник. Узнаешь в школе в 7 классе.
ясно, еще один рваный инцел
Аноним 12/10/22 Срд 22:21:57 #27 №566246 
>>566244
>пруфы
Ты утверждаешь - ты пруфай.
>ясно, еще один рваный инцел
Без проекций. То что тебя рвёт с полового отбора - твои проблемы.
Просто и молча (в смысле молча, орать же нужно и кряхтеть), чтобы рандом тяночке присунуть потому что она не против, будет при коммунизме.
Сейчас сосай бибу и наслаждайся отношениями уровня животного мира, когда ты должен пройти половой отбор самок.
Аноним 12/10/22 Срд 23:10:29 #28 №566248 
>>566246
>пруфов нет
>Без проекций
>ти по рвался
Аноним 13/10/22 Чтв 06:39:29 #29 №566256 
>>566248
>пруфы
Ты утверждаешь - ты пруфай.
Аноним 13/10/22 Чтв 07:13:16 #30 №566257 
>>566256
>Не бывает такого.
Это ты утверждаешь, аутист
>Половой отбор называется, порвак-младшекласник. Узнаешь в школе в 7 классе.
Такому долбоебу бессмысленно что-либо пруфать. Пятак обссать и ремня отцовского дать, чтоб не дерзил
Аноним 13/10/22 Чтв 07:53:20 #31 №566258 
>>566257
>Это ты утверждаешь, аутист
@
>>566226
>одни животные просто и молча трахают друг друга, и идут по своим делам, забыв об этом.
А ты немного туповат, верно?
Аноним 13/10/22 Чтв 09:32:59 #32 №566261 
>>566258
Не понимаю микропук дурачка. Разверни мыслю
Аноним 13/10/22 Чтв 12:33:43 #33 №566266 
>>566225
Т.е. проблема только в способности выносить, а зачать скорее всего реально?
Аноним 13/10/22 Чтв 12:34:37 #34 №566267 
>>566207
Хотел еще одну инвалидность получить?
Аноним 13/10/22 Чтв 14:54:18 #35 №566274 
Почему Ян Кёртис повесился,а вы ещё нет?
Аноним 13/10/22 Чтв 21:30:37 #36 №566279 
>>566266
Мало данных. У нас же нет расшифрованного генома австралопитека. Но многие виды, разошедшиеся в эволюционном древе всего лишь 2-3 млн лет назад, способны скрещиваться. Но даже если гибридизация возможна, не факт, что потомство было бы плодовитым и вообще полноценным.
Аноним 13/10/22 Чтв 22:42:02 #37 №566280 
>>566231
Так же как и ленивцы, майские жуки и прочие беззащитные создания. Их хвосты в принципе служат отвлекающим и дезориентирующим устройством. Для их ареала обитания этого достаточно.
Аноним 13/10/22 Чтв 23:07:06 #38 №566282 
Вопрос не научный, но фундаментальный. Почему вы не сходите с ума? Я имею в виду тех из вас, кто учёный или просто обладатель очень сильно развитых лобных долей мозга, интеллекта в целом, не страдающих при этом депрессией. Что даёт вам уверенность в необходимости существования, желание бороться за выживание и осмысленно размножаться, искать способы изучения и решения проблем? По сути идёт борьба с упорядочиванием энтропии в стабильные и относительно безопасные системы. Но если уже теории говорят от тепловой смерти Вселенной или эпохи чёрных дыр, в чём смысл лично ваших жизней, которые даже не микробы в сравнении с масштабами происходящего, чьи продолжительности жизней и познания ограничены микроскопическими отрезками времени? Как вы справляетесь с энтропией в виде гопников, мобилизации, эпидемии. Я хочу узнать есть ли логическое и научное объяснение жизни у вас, или всё всегда и для всех сводится к банальной работе гормоналки с позитивным темпераментом, которые и задают био-химические процессы в организме, направляя мышление и психику по рельсам из пункта А в пункт Б без остановок. А всё остальное получается дефектным и подлежит утилизации эволюцией.
Аноним 13/10/22 Чтв 23:26:06 #39 №566285 
>>566226
>Почему
Потому что так сложилось исторически.
В теории эволюции нет внятных математических моделей, и по-видимому быть не может. Поэтому корректного ответа на вопрос "почему так, а не иначе?" получить не выйдет.

Все эти досужие домыслы задним числом в стиле эволюционизма по сути лженаучны.
Нужно просто констатировать, что жизнь стремится использовать весь спектр возможностей.
Аноним 14/10/22 Птн 05:51:07 #40 №566293 
>>566285
>Потому что так сложилось исторически
Так можно обо всем на свете, не изучая ничего, типа умный. Нам надо выяснить почему, чтобы управлять этим и понять самих себя
Аноним 14/10/22 Птн 05:52:31 #41 №566294 
>>566212
Порезать бутылку на полоски и засунуть в 3д-принтер
Аноним 14/10/22 Птн 13:43:21 #42 №566308 
Привет! А есть ли тред по генетике и происхождению? Очень интересно пообщаться на тему гаплогрупп, переселений народов, сайтов. куда можно вбить свои данные и посмотреть и т.д.
Аноним 14/10/22 Птн 16:30:02 #43 №566311 
>>566261
А ты очень туповат. Давай ещё раз.

Именно ТЫ утверждаешь что у какой-то части организмов нет полового поведения. Так приведи примеры этих организмов.

Ты же не поехавший долбоебикс-порвак-инцел что придумал эти организмы без полового поведения?
Аноним 14/10/22 Птн 16:38:31 #44 №566312 
>>566308
сюда пиши https://2ch.hk/sci/res/566122.html
Аноним 14/10/22 Птн 18:36:44 #45 №566315 
>>566311
Ахахаха, тупая опущенная членодевка.
99% живых организмов просто насилуют самок и идут дальше.
Остальные вырождаются и умирают, поэтому их всего где-то 1%.
Аноним 14/10/22 Птн 18:59:38 #46 №566318 
>>566315
Пруфы-то будут или так и будешь шизофантазировать хуиту?
Аноним 14/10/22 Птн 19:11:50 #47 №566319 
>>566318
Пруфы на то что ты куколд-хуесоска промытая?
Аноним 14/10/22 Птн 19:39:35 #48 №566320 
>>566319
> шизик понял что обосрался и ушёл в жопоболь и отрицание
Ясно.
Аноним 15/10/22 Суб 19:04:33 #49 №566336 
>>566320
Ты чего порвался, куколд-хуесосочка?
Аноним 15/10/22 Суб 22:23:46 #50 №566350 
>>566285
Про популяционную статистику ты видно ничего не слышал
Аноним 15/10/22 Суб 22:25:06 #51 №566351 
16655974119250.gif
>>566163 (OP)
Коллапс волновой функции это физическое явление или математическая абстракция для удобства описания?
Аноним 15/10/22 Суб 22:32:46 #52 №566352 
>>566226
Насколько я знаю, у популяционной статистики точного ответа еще нет

Могу выдать свое предположение:
Есть эксперимент с двумя популяциями ярких рыбок одного вида из одной популяции размещенными соответственно в двух разных басейнах. В одном есть хищники (группа А), а другом - нет (группа B)
Через определенное количество поколений группа А становилась более тускло окрашенной, а группа B сохраняла/приобретала яркий окрас
Вот тебе и естественный и половой отбор
Аноним 15/10/22 Суб 22:37:56 #53 №566353 
>>566351
Физическое явление, волнообразное тело берет и исчезает, независимо от его размера.
Аноним 15/10/22 Суб 23:21:52 #54 №566354 
>>566351
Это даже не абстракция, т.е. отвлечённое описание, а чисто математический прием.
Аноним 15/10/22 Суб 23:39:31 #55 №566355 
>>566354
То есть с детерминизмом все в порядке?
Аноним 15/10/22 Суб 23:43:48 #56 №566356 
>>566163 (OP)
Поясните примитиву как можно измерить, и как меряли, критическую плотность Вселенной для определения ее замкнутости или бесконечности. Как вообще тут логика определения A из B работает
Аноним 16/10/22 Вск 02:18:55 #57 №566358 
1665875935976.png
>>566163 (OP)
Не знаю, где тут это запостить (в химии треде?), пусть будет здесь.
Мне (гениальному писателю) надо построить модель замкнутой биосферы. Планета полностью изолирована от космического излучения, потери тепла компенсируются теплотой из недр. Планета старая, фотосинтеза нет. Хемосинтеза - уже тоже нет миллиард лет. Сожрали все минералы, короче. Каким образом на ней будет возможна процветающая биосфера? Вряд ли на равномерно разогретом шаре без легкодоступной нямки будет расти что-то большее, чем пара бактерий.

Пока лучшее мое предположение: источник жизни - биомы вокруг глубоководных разломов и горячих точек. Там есть локальный разогрев выше средней по больнице и приток вещества из мантии. Так вот, помыслил я, что обитающие там формы жизни и хемосинтезом пробавляются, и каким-нибудь термоэлектричеством за счет перепада температур. На поверхность моря восходят потоки органики (через десять желудков консументов и прочих сапрофитов) и воды с повышенной кислотностью.

Теперь самое сложное, каким образом при этом процветает суша. Допустим, она заросла десятиметровым слоем гидрофильного "мха", который буквально пьет обогащенную органикой кислотную воду из моря и закачивает ее вглубь суши. При этом есть и пресные атмосферные осадки. Получается перепад в кислотности поступающей воды.

Таким образом, море работает на синтез органики и повышение кислотности. Суша работает на разрушение органики, связывание азота и выщелачивание воды, которая вернется с "компостом" в глубоководные биомы и даст уже там нужные перепады величин. Получается круговорот веществ, который, одновременно, утилизирует со всем возможным КПД энергию радиоактивного ядра планеты.

Такие дела. Я бы яснее расписал, но как-то спонтанно сюда пришел. На самом деле, я уже в пижамке.
Аноним 16/10/22 Вск 02:19:42 #58 №566359 
>>566351
После измерения волновая функция меняется. Значит физическое.
Аноним 16/10/22 Вск 04:19:54 #59 №566360 
Почему чайник (не электрический, хотя что тот, что тот) в процессе закипания потрескивает, в принципе шумы издает, а закрытая кастрюля нет?
Аноним 16/10/22 Вск 04:21:02 #60 №566361 
>>566165
>>566168
Видел эту хуйню на реддите, там объяснили что под платформой верхней колесико ускоряющее шарик и работать будет пока не сядут батарейки.
Аноним 16/10/22 Вск 09:26:27 #61 №566366 
>>566360
Накипь отлетает.
Аноним 16/10/22 Вск 11:07:02 #62 №566369 
>>566351
Строго говоря ни то, ни другое.
Коллапс это актуализация строго одного состояния из всего множества квантовых состояний. Как это именно происходит вынесено за скобки и никак не рассматривается в квантовой теории, вместо этим занимаются маняфилософы.
И физическим явлением его нельзя считать, поскольку нам принципиально недоступны квантовые состояния. О них мы можем судить только ретроспективно из интерпретации набора актуализированных состояний.
Аноним 16/10/22 Вск 11:40:09 #63 №566370 
>>566163 (OP)
Как минимум во сколько раз человеческая кожа должна быть прочнее по всем параметрам, чтобы выдерживать попадание снаряда из ручного, но стационарного огнестрела крупного калибра, и при этом по большей части защитить внутренние органы и кости?
Игнорируя остальные физические законы конечно же
Аноним 16/10/22 Вск 11:51:51 #64 №566371 
>>566370
Приблизительно, разумеется. Никто голову ломать и считать средние характеристики кожи не будет. Кожа на пузе, например.
Аноним 16/10/22 Вск 12:34:37 #65 №566375 
>>566370
Мелкий и средний калибр и металлы то пробивает. Если ты супер-герое-комиксист, либо забей на реализм, либо на комиксы
Аноним 16/10/22 Вск 13:02:46 #66 №566377 
>>566369
>Как это именно происходит вынесено за скобки и никак не рассматривается в квантовой теории
Вообще-то, вполне рассматривается. Классические системы это частный случай квантовых, поэтому взаимодействие классического детектора и квантового объекта, квантовая механика описывает без всяких проблем. Это буквально лежит в её границах применимости
Аноним 16/10/22 Вск 13:09:20 #67 №566378 
>>566375
Какие металлы? Марганец,? Сталь из макарыча не пробивается, сколько в одну точку не стреляй.
Аноним 16/10/22 Вск 14:41:34 #68 №566385 
>>566378
https://youtu.be/tasYwbsm6XA
Аноним 16/10/22 Вск 15:26:53 #69 №566387 
>>566366
/sci/ на острие науки блять. Почему шумит чайник нагуглить не сложно, там хитровыебанная система с шариками воздуха которые схлопываются в верхних холодных частях воды, создают вакуум и собственно шумят, а вот почему кастрюля так не шумит.
Аноним 16/10/22 Вск 16:26:42 #70 №566394 
>>566387
Не та акустика. Чайник здесь как саксофон, а кастрюля как труба
Аноним 16/10/22 Вск 16:28:49 #71 №566395 
>>566394
У меня еще были мысли на толщину стенок, более равномерное прогревание, но да, и это возможно.
Аноним 16/10/22 Вск 19:04:31 #72 №566406 
Объясните, пожалуйста, что значит влажность воздуха? В интернете только какие то научные формулы и объяснения. Простыми словами можно? Типа на что конкретно она влияет и для чего нужны всякие осушители и увлажнители?
Аноним 16/10/22 Вск 19:19:42 #73 №566408 
>>566406
количество воды в воздухе
повышает теплопроводность, может высасывать воду из организма
Аноним 16/10/22 Вск 21:39:55 #74 №566412 
Вредно ли систематическое употребление сала?
Аноним 16/10/22 Вск 22:15:45 #75 №566413 
>>566412
Схохлишься.
Аноним 16/10/22 Вск 22:52:01 #76 №566417 
>>566413
Я и так беларус. Тут много полухохлов
Аноним 17/10/22 Пнд 11:11:15 #77 №566431 
>>566412
Оно довольно калорийно, есть шанс ожиреть и мутировать в скуфа. Очень вредно жрать с алкоголем.
Аноним 17/10/22 Пнд 20:03:12 #78 №566444 
Как социологии доказывают валидность соц. опросов? Например телефонных? Прочитывал несколько статей и нихуя не понял.
Типа, у нас есть методы получить хорошую выборку, несмотря на то, что базы номеров мы получаем хуйпойми как, 80% не берут трубку, 90% отказываются пройти все этапы опроса и поэтому их ответы не учитываются. Это же наебалово, да?
Аноним 17/10/22 Пнд 20:05:49 #79 №566445 
>>566431
>Очень вредно жрать с алкоголем.
Нет. Алкоголь особенно вреден с сахарами.
>Оно довольно калорийно
Калорийность это количество тепла, выделяемого при горении в чистом кислороде, держу в курсе.
Аноним 17/10/22 Пнд 21:40:47 #80 №566447 
>>566445
>Калорийность это количество тепла, выделяемого при горении в чистом кислороде, держу в курсе.
Это так. Но к чему ты этот тривиальный факт решил здесь упомянуть?
Любая калорийность, указываемая на упаковке пищевого продукта, определяется именно так.
Аноним 17/10/22 Пнд 21:42:43 #81 №566448 
>>566444
Да.
Социология - лженаука. Но, тем не менее, в стационарных условиях соцопросы дают воспроизводимые и достаточно надёжные результаты.
Аноним 17/10/22 Пнд 21:51:55 #82 №566450 
>>566448
>в стационарных условиях
Это каких?
>Социология - лженаука
>соцопросы дают воспроизводимые и достаточно надёжные результаты
шта
И меня интересуют объяснение с т.з. социологов. Каким образом они рассчитывают считать сигнал, пошедший через столько искажений? Это же тривиальный вопрос, он должен быть объяснен в первую очередь в любом популярном тексте про опросы. Но нет.
Аноним 17/10/22 Пнд 21:57:03 #83 №566451 
>>566447
К тому, что калорийность продуктов слабо связано с тем, толстеют от них, или нет.
Аноним 17/10/22 Пнд 22:26:13 #84 №566452 
>>566451
Связана самым прямым образом. Но с необходимыми оговорками, касающимися усвояемости продукта.
Бензин никак не усваивается, а сало здоровым взрослым человеком - прекрасно.
Аноним 17/10/22 Пнд 22:30:00 #85 №566453 
>>566452
И какая же "усвояемость" у сала в численном выражении?
Аноним 17/10/22 Пнд 22:37:26 #86 №566454 
>>566450
>Это каких?
Таких, когда внешние условия меняются медленно.
>шта
Та.
>>566450
>Каким образом они рассчитывают считать сигнал
Ну тут горы литературы наворочены. Статистические методы позволяют все это учесть, но 99% социологов ими пользоваться не в состоянии.
См., например, статью и список литературы к ней
https://www.sostav.ru/blogs/32546/7113
Аноним 17/10/22 Пнд 22:49:13 #87 №566455 
>>566163 (OP)

Несколько лет назад, пока в Европе измеряли миллиньютоны на стендах и чесали репы, китайцы сказали что выкинут в космос настоящий em-drive и посмотрят по факту работает или нет.
Так вот.
Чо там у китайцев?
Аноним 17/10/22 Пнд 22:53:13 #88 №566457 
>>566453
>И какая же "усвояемость" у сала в численном выражении?
Ишешь раскладку пищевого продукта на основные нутриенты и читаешь биохимическую и физиологическую литературу по каждому.

На 100 г сала приходится:
Жиры — 88,69 г,
Белки — 2,92 г,
Вода — 7,69 г,
Зола — 0,70 г.

Раскладку по жирным кислотам можно посмтреть, например, тут https://fitaudit.ru/food/129041/fat

Всё это усваивается практически на 100 %, поэтому каждый съеденный кусок сала привносит в организм столько калорий, сколько указано в справочнике. Если этот приток калорий не будет скомпенсирован соответсвующим расходом, организм будет жиреть.
Конечно, можно сожрать разом 3 кг сала, получить расстройство ЖКТ и снизить усвояемость. С этим никто не спорит.
Аноним 17/10/22 Пнд 22:53:15 #89 №566458 
>>566454
>внешние условия меняются медленно
шта
штааааааа
>Статистические методы позволяют все это учесть
Чел,
>80% не берут трубку, 90% отказываются пройти все этапы опроса и поэтому их ответы не учитываются
Как "статистические методы" позволяют это учесть? В статье ничего про это нет.
Аноним 17/10/22 Пнд 22:53:43 #90 №566459 
>>566455
Не работает.
Аноним 17/10/22 Пнд 22:55:43 #91 №566460 
>>566457
>каждый съеденный кусок сала привносит в организм столько калорий
чего приносит лол
тепла от горения в кислороде?
>Всё это усваивается практически на 100 %
что блядь "усваивается"
Аноним 17/10/22 Пнд 22:57:03 #92 №566461 
>>566458
На хуй иди. Или читай то, на что ссылается статья. Или приведи свой список прочитанного и объясни, почему все это - говно.
Аноним 17/10/22 Пнд 22:58:07 #93 №566462 
>>566163 (OP)
Это вечный двигатель?
Аноним 17/10/22 Пнд 22:58:29 #94 №566463 
>>566460
Ты реально не понимаешь, что при превращении стеариновой кислоты в углекислый газ и воду в любых условиях выделится одинаковое количество энергии?
Аноним 17/10/22 Пнд 23:02:26 #95 №566464 
>>566463
жиры, поступающие в организм
1. не все превращаются в углекислый газ и воду
2. на все способы превращения тратится "энергия" от других превращений
Аноним 17/10/22 Пнд 23:05:04 #96 №566465 
>>566461
Ты якобы знаешь о чём говоришь, вместо этого отсылаешь к книгам, указанных в нерелевантных примечаниях.
>приведи свой список прочитанного
Я нихуя не читал, поэтому задаю простой вопрос. Ответа ты не знаешь, ОК.
Аноним 18/10/22 Втр 15:40:45 #97 №566498 
Подскажите, куда копать чтобы понять, какую массу и на какую высоту может поднять ракетный двигатель на определенном ракетном топливе в определенном количестве? Например на сахарном топливе 10-15 кг на высоту 2-3 км. Как это прикидывать примерно?
Аноним 18/10/22 Втр 16:47:54 #98 №566500 
У плоскоземельщиков есть одно положение, что гравитации не существует, а всё дело в плотности. Мяч падает, потому что плотнее воздуха, Титаник тонет, потому что тяжелее воды.
Как это опровергнуть? Может, теоретически (найти противоречие в этой модели) или эмпирически (найти несоответствие реальному миру)? Чтобы не вдаваться в длинные дискуссии, один меткий удар.

Навскидку, есть пример, что мяч не падает или жидкости не смешиваются на борту МКС, но адепты не признают космос.
Мысленный эксперимент с падающим лифтом по идее работает. Там ты будешь висеть в воздухе в одной точке и не падать внутри лифта. Но таким людям вряд ли подойдут мысленные эксперименты.

Что на твой взгляд подойдёт без натурных экспериментов, анон?
Аноним 18/10/22 Втр 16:49:30 #99 №566501 
>>566498
Kerbal space program
Аноним 18/10/22 Втр 17:32:28 #100 №566503 
>>566500
> Как это опровергнуть?
Утопление более плотного в менее плотном работает только благодаря гравитации. Например, физический смысл архимедовой силы в том, что под действием гравитации атомы среды давят друг на друга (чем ниже, тем больше давление), и так на погруженное тело действует снизу в верх градиент давления, который может компенсировать или превысить вес (mg) погруженного тела. А может и не превысить, есть средняя плотность тела больше плотности среды. Тогда тело тонет - под действием гравитации, блджадь!
Аноним 18/10/22 Втр 17:36:50 #101 №566506 
>>566500
Пускай сами доказывают почему более плотный тонет в менее плотном. Пускай без гравитации расскажут что такое более плотный а что такое менее плотный.
Аноним 18/10/22 Втр 17:40:56 #102 №566507 
>>566498
Если идеальный случай рассматривать, то по энергии сгорания топлива, плюс нужна температура сгорания, через температуру сгорания как-то вывести условный кпд, через скорость истечения, чем больше скорость истечения тем выше кпд
И потом через mgh, сравнивая с энергией сгорания умноженной на этот высчитанный кпд, выражать h-высоту.
Но это в идеальных условиях, в реальных условиях ты даже примерно не посчитаешь сколько получится, с реальным двигателем, нужны слишком сложные расчёты и модели, ты не сделаешь.
Аноним 18/10/22 Втр 17:43:24 #103 №566508 
>>566500
Как выше сказали, у версии плоскоземельщиков просто нет физического смысла. Мяч падает, потому что плотнее воздуха. Подчеркнутое - недоказанное утверждение, к которому надо бы прилагать объяснение: а схерали более плотное падает? Гравитация дает объяснение, физический смысл, как падение связано с плотностью. А исключение гравитации превращает утверждение в набор слов.
Аноним 18/10/22 Втр 18:42:33 #104 №566513 
SeloIdoiki.JPG
Почеум было любит дойки?
Аноним 18/10/22 Втр 18:51:40 #105 №566514 
15262489812640.jpg
>>566163 (OP)
Анончики, поясните мне за ситуацию:
в свободное время я занимаюсь лепкой, моё хобби, получаю от этого удовольствие. Когда я начал принимать антидепрессант, флуоксетин, желание лепить что то пропало. Желание каким то творчеством заниматься пропало, хоть и настроение поднялось сильно.
Как это работает? Почему хорошее настроение убивает творческий потенциал?
Аноним 18/10/22 Втр 19:06:07 #106 №566516 
>>566514
Художник должен страдать. Муза - хтонический монстр, питающийся страхом и болью.
Аноним 18/10/22 Втр 19:15:29 #107 №566517 
>>566514
/psy
Аноним 18/10/22 Втр 19:39:18 #108 №566518 
>>566517
Там только пропагандисты пыни сидят, врачей там нет.
Да и среди психиатров и психологов практически нет врачей, одни шизики религиозные, 99% где-то.
Аноним 18/10/22 Втр 22:35:40 #109 №566534 
>>566517
У меня скорее интерес обычный, а не попытка вылечить депру.
Аноним 19/10/22 Срд 08:42:53 #110 №566546 
>>566516
Этот прав.
Желание творить не может возникнуть когда нет делания изменить мир.
А когда ты лежишь слюни пускаешь в экстазе желание что-то менять в принципе не возникает.
Аноним 19/10/22 Срд 08:48:14 #111 №566547 
>>566513
Потому же почему и ярко выраженную талию и широкий таз.
Плоское бревно у подкорки не ассоциируется со здоровым потомством.
Аноним 19/10/22 Срд 19:13:32 #112 №566560 
image.png
Почему самолет в падении теряет управляемость? Как я понял, это происходит только в 1 случае из за того, что крыло находится под неадекватным углом атаки к потоку воздуха, так как скорость направлена вниз, крыло куда-то вдоль горизонта выходит по прикидке угол атаки градусов 100-90. А вот если как на ситуации справа постепенно направлять нос вниз, чтобы хорда крыла и направление скорости, а значит и потока воздуха (с незначительной поправкой на ветер), постоянно были парралельны друг-другу, то никаких проблем в управлении быть не должно, отличных от нормальных условий? Ну кроме тех, что от высокой скорости. Они бы появились и так, летя ты горизонтально на большой скорости.
Единственное, может быть, что отличается ситуацию, так это повышенные перегрузки из за центробежной силы, но кроме как на большее усилие приводов закрылок каких-нибудь это же не должно возыметь эффекта?
Аноним 20/10/22 Чтв 12:18:22 #113 №566568 
16661669223640.png
>>566513
Потому что.
Аноним 21/10/22 Птн 02:30:22 #114 №566585 
Почему науку считали и считают хуетой?
Всем похуй на бессмертие, космос и остальное
Аноним 21/10/22 Птн 02:33:49 #115 №566586 
>>566585
Большинство почему-то не замечает изменений.
Аноним 21/10/22 Птн 06:57:43 #116 №566588 
>>566586
А если изменений нет - значит наука не работает.
Аноним 21/10/22 Птн 08:07:04 #117 №566589 
>>566588
Просто большинство не видит деревьев стоя посреди леса
Аноним 21/10/22 Птн 17:16:21 #118 №566598 
>>566560
> скорость направлена вниз, крыло куда-то вдоль горизонта
В этом случае подъемной силы нет и не работают нормально элементы управления, так как их вообще не с тех сторон обдувает.
> то никаких проблем в управлении быть не должно, отличных от нормальных условий?
Ты только что пикирующий бомбардировщик.
Аноним 21/10/22 Птн 17:18:35 #119 №566599 
>>566589
Ничо, сейчас "деревья" вырубят, вот и узнаем, каково это жить без достижений науки.
Аноним 21/10/22 Птн 18:03:28 #120 №566601 
>>566163 (OP)
Биологическая задча продолжения рода = самое важное, что ест у нас?
унас нет бессмертия
но есть его форма в виде почкования/создания своей копии и клониования себя тким образом.


вот у пауков есть созаннеи? черная вдова-самка кущаюшая самца после спавривания - она же не чувсвуте угрызений совести а он не чувствует боли и сожалений? они просто рады выполнить задачу, к выполнению которой их подтолкнул глубокий инстинкт, и похуй на всё?
Аноним 21/10/22 Птн 19:20:07 #121 №566603 
>>566601
> Биологическая задча продолжения рода = самое важное, что ест у нас?
Да.
Не предохраняйся, живи как животное, как задумано природой, пусть твоя свиноматка рожает каждый год, один за другим, сколько сможет за жизнь, выполняй самую важную биологическую задачу.
Аноним 21/10/22 Птн 21:35:32 #122 №566605 
>>566601
Размножение это задача возникающая на определенном этапе. У какого-нибудь инцела самое важное это его работа, которая приносит массу пользы обществу и тем кто заводит детей. Как у муравьев
Аноним 21/10/22 Птн 22:22:52 #123 №566606 
r
Аноним 21/10/22 Птн 22:26:43 #124 №566607 
ФотонныйдвигательЗенгера.jpg
Провожу мысленные испытания фотонного двигателя. Взял магнетрон и отражатели. Установил все на тележку. Вбухиваю в магнетрон целый киловатт энергии. Эта херня никуда не едет. Что я делаю не так?
Аноним 21/10/22 Птн 22:38:34 #125 №566608 
Чё когда там уже термояд запустят? Нам уже 70 лет обещают, что вот через 30 лет у нас будет бесплатная термоядерная энергия, но этого не происходит
Аноним 21/10/22 Птн 22:48:04 #126 №566610 
Чем вы отличаешься от верунов? Верите в фантазии ебаклаков которые ошиблись в расчетах и придумали несуществующие костыли типа темной хуйнэргии, которую никак не увидеть, но она есть, ыыыы, чтобы их кривые расчеты хоть как то сводились.
Аноним 21/10/22 Птн 22:51:50 #127 №566611 
1666381909646.jpeg
>>566608
> Нам уже 70 лет обещают
Это точно! Pic.related совецкая научпоп книга 1959 года!
Аноним 21/10/22 Птн 23:08:27 #128 №566612 
>>566610
Ничем.
Верующий даже подольше проживёт чем ученный
Аноним 22/10/22 Суб 01:08:46 #129 №566615 
>>566612
Верующий потому верующий, что для него основополагающий - сам факт смерти в сколь угодно отдаленном будущем. Поэтому он сходит с ума заранее.
Аноним 22/10/22 Суб 17:41:51 #130 №566636 
129331009892097611341373711612041099081913n.png
Пикрил. Если всё давно известно, почему бы просто не колоть по расписанию гормоны для создания того или иного эмоционального фона, или создания рефлекса на что-либо? Обучение, например. Или напротив, если плохой эмоциональный фон - принудительно его повышать.
Аноним 22/10/22 Суб 19:55:25 #131 №566643 
image.png
Что происходит в этой схеме? Объясните момент: если по определению ток течет туда, где меньший потенциал, то как ток может течь через источник от + к -, ведь по определению потенциал на + клеме больше на ЭДС.
Аноним 22/10/22 Суб 21:46:06 #132 №566646 
>>566643
4E побеждают 1Е
в источнике хитрая реакция поддерживает разницу потенциалов
Аноним 22/10/22 Суб 22:09:57 #133 №566650 
>>566636
Умозрительная хуита уровня определение состояния компьютера по тому как вертятся его кулеры.
В различных частях мозга и вообще органах различное соотношение гормонов. Вдобавок гормоны мы как быть связаны с рецепторами или транспортными, белками так и просто свободно плескаться в ликворе или крови.
На пике походу просто показывается что находится в свободном виде в крови без учета всего остального. В организме дохуя обратных связей и конкурирующих механизмов. Если ты повторишь внешних фон, без учета их получишь совершенно другое или даже сломаешь тонкие механизмы.
Аноним 22/10/22 Суб 22:15:53 #134 №566651 
>>566163 (OP)
Продвинутая квантовая механика, это реально?
https://www.researchgate.net/publication/356491520_Quantum_time_inversion_to_prevent_DDoS_attacks_A_potentially_realizable_TENET_technology
Аноним 22/10/22 Суб 22:32:00 #135 №566652 
>>566636
Так уже делали, например в фильме эквилибриум.
Аноним 22/10/22 Суб 22:35:19 #136 №566653 
>>566607
Фалкон Маска сжигает 200 тонн керосина летя до орбиты, хз сколько секунд, минуты три вроде.
За 3 минуты 200 тонн.
Если бы там был идеальный преобразователь тепла в свет-это была бы тяга 200-300кг
А теперь сам посчитай разницу в энергии 200тонн керосина за 3 минуты и твоего 1кВт.
Аноним 23/10/22 Вск 06:48:12 #137 №566656 
>>566607
>Что я делаю не так?
Твоя основная ошибка в твоём генетическом коде.
Аноним 23/10/22 Вск 06:49:21 #138 №566657 
>>566599
>сейчас "деревья" вырубят

Когда будет срублено последнее дерево, когда будет отравлена последняя река, когда будет поймана последняя птица, — только тогда вы поймёте, что деньги нельзя есть.
Аноним 23/10/22 Вск 08:17:14 #139 №566658 
>>566172
>квази-?
>>566178
>Типа наподобие, похожее
или кажущееся
Аноним 23/10/22 Вск 08:20:00 #140 №566659 
>>566163 (OP)
Вопрос: Как дэгэнэраты становятся успешноблядями?

Пример. Многоденежный автор научных новостей пишет такое:
"Ранее реактор HL-2M Tokamak, построенный в городе Чэнду провинции Сычуань, смог разогреть плазму до 150 млн °C, что в 10 раз больше, чем в ядре Солнца. Реакция поддерживалась 10 с. Судя по всему, речь идёт о температуре нагрева электронов в плазменном жгуте. Для запуска устойчивой термоядерной реакции требуется нагреть до температуры 100 млн °C ионы газа в плазме, которые в два раза тяжелее электронов. Это означает, что разница температур между электронами и ионами будет примерно двукратная. Иначе говоря, реактор нагрел ионы до 75 млн °C и должен повысить это значение ещё на 25 млн °C."
Источник:https://3dnews.ru/1076182/noviy-kitayskiy-reaktor-priblizilsya-k-ustoychivoy-termoyadernoy-reaktsii
Аноним 23/10/22 Вск 08:25:18 #141 №566660 
image.png
>>566646
Скорее стремиться тогда сделать на + клемме на ЭДС больше? А как не может, например из за другого сильного источника, то просто сопротивляется? Вот как-то так. Схема без резистора че то не собиралась
Аноним 23/10/22 Вск 08:59:35 #142 №566661 
>>566657
Деревья возобновляемый ресурс
Аноним 23/10/22 Вск 10:23:12 #143 №566662 
>>566661
Чернозем тоже. Только формирование зрелых почвенных профилей с полноценными гумусовыми горизонтами - исключительно длительный процесс, продолжающийся в течение столетий и тысячелетий. Уничтожение же происходит на порядок быстрее.
Аноним 23/10/22 Вск 12:53:12 #144 №566664 
>>566662
Лихо ты от умирающих деревьев к созданию чернозёма перепрыгнул
Аноним 23/10/22 Вск 15:56:55 #145 №566666 
image.png
Напомните, почему пикрелейтед не работает?
Аноним 23/10/22 Вск 15:59:13 #146 №566667 
>>566666
Скорость причинности не позволяет.
Аноним 23/10/22 Вск 16:04:37 #147 №566668 
00.jpg
>>566664
Странно, что ты не видишь всей картины.
Потому что твой тезис про возобновляемый ресурс опирается на то, что деревьям есть где расти.
Аноним 23/10/22 Вск 16:05:49 #148 №566669 
>>566666
Сумма векторов действующий сил равна нулю.
Аноним 23/10/22 Вск 16:11:07 #149 №566670 
>>566664
Собственно, он даже прав. Лес - это не сумма деревьев, а экосистема, куда входит куча видов всех царств, климат и почва. То есть, для создания леса с нуля нужно знатно поработать.
Аноним 23/10/22 Вск 16:17:57 #150 №566671 
>>566670
>для создания леса с нуля
На текущем этапе развития науки человечества с нуля невозможно ничего.
Более того, вся человеческая цивилизация в текущем её виде не может существовать без невозобновляемого выкапываемого сырья.
Все разговоры про создание с нуля, это бред умалишенного, который думает, что электромобили экологически чистый транспорт, потому он никогда не считал всю цепочку производства и не учитывал всего того говна, которое просто выливается в окружающую среду, на пути этого производства.
Аноним 23/10/22 Вск 17:44:22 #151 №566672 
>>566282
>Что даёт вам уверенность в необходимости существования
Лично у меня нет необходимости, но есть интерес. Интересно посмотреть, чем это всё закончится.
Аноним 23/10/22 Вск 17:47:20 #152 №566673 
>>566315
>99% живых организмов просто насилуют самок
Самка богомола в обнимку с чёрной вдовой в ужасе смотрят на тебя.

Ещё вопрос, кто кого насилует, божественные тяночки или бесполезные хуеносцы.
Аноним 23/10/22 Вск 17:49:35 #153 №566674 
>>566352
>Через определенное количество поколений группа А становилась
К сожалению, попытки воспроизвести результат эксперимента провалились. Окраска самцов не зависит от давления среды.
Аноним 23/10/22 Вск 18:29:53 #154 №566679 
>>566282
Желание станцевать на могилах всех сук и мудаков, конечно же.
Аноним 24/10/22 Пнд 11:49:33 #155 №566705 
Человек чувствует что задыхается когда количество углекислого газа в крови превышает определенный процент. Если человек будет дышать смесью в которой кислорода столько же сколько обычно, а все остальное углекислый газ, то как он будет себя чувствовать? Будет все время мучится и задыхаться хотя кислорода ему будет хватать для жизни или же будет дышать нормально?
Аноним 24/10/22 Пнд 13:05:25 #156 №566711 
>>566705
Концентрация углекислого газа свыше 8% смертельна.
Аноним 24/10/22 Пнд 14:10:01 #157 №566712 
>>566705
>кислорода ему будет хватать для жизни
Это не так работает, эффективность газодиффузионных мембран зависит от разности концентраций.
Аноним 24/10/22 Пнд 14:52:10 #158 №566717 
>>566712
То есть, если поднять давление воздуха в два раза, то предельную концентрацию СО2 надо снижать?
Аноним 24/10/22 Пнд 15:07:16 #159 №566719 
>>566717
Для чего "надо"?
Аноним 24/10/22 Пнд 15:30:00 #160 №566722 
>>566719
Ну, я хз, для чего. Чтобы не было душно и мозги не тормозили от углекислоты?
Аноним 24/10/22 Пнд 15:49:03 #161 №566725 
представьте что у меня есть шприц х который по проводнику толкает раствор со скоростью 5 мл ч, и шприц у со скоростью 15 мл ч, оба они подключены к одному проводнику - скорость в нём будет 10 мл ч?
Аноним 24/10/22 Пнд 20:04:01 #162 №566730 
>>566163 (OP)
Обратите внимание как шарики дергаются друг от друга и от лунки.
Вот вам и ответ.
Аноним 24/10/22 Пнд 21:01:42 #163 №566732 
1666634503428.png
С какой скоростью портится урановое топливо? Например, энти топливные таблетки UO2 с пикчи? Если ТВС после изготовления проваляется без дела тысячу лет, она еще будет пригодна для использования?
Аноним 24/10/22 Пнд 21:31:13 #164 №566733 
Если будут вне внешнего нейтронного потока и рядом не будет отражателей и замедлителей нейтронов, то с таблетками ничего не будет происходить сотни миллионов лет. Уран даже обанощенный сам по себе слаборадиактивный.
Аноним 24/10/22 Пнд 22:37:15 #165 №566740 
>>566282
> Почему вы не сходите с ума?
Не хочу.

> Что даёт вам уверенность в необходимости существования, желание бороться за выживание и осмысленно размножаться, искать способы изучения и решения проблем?
Стремления + немного философии Ницше, конкретно про человека трудящегося.

> в чём смысл лично ваших жизней, которые даже не микробы в сравнении с масштабами происходящего, чьи продолжительности жизней и познания ограничены микроскопическими отрезками времени?
У жизни нет абсолютного смысла, но есть относительный, но как ты понимаешь содержание такого смысла в каждый момент времени может отличаться от предыдущего.

> Как вы справляетесь с энтропией в виде гопников, мобилизации, эпидемии.
Когнитивно-поведенческая психология.

> Я хочу узнать есть ли логическое и научное объяснение жизни у вас, или всё всегда и для всех сводится к банальной работе гормоналки с позитивным темпераментом, которые и задают био-химические процессы в организме, направляя мышление и психику по рельсам из пункта А в пункт Б без остановок.
Объяснение жизни? Гугли первый РНК-организм, думаю поймешь, что смысла в биологической жизни как такового нет и это просто химический самоповторяющийся генератор. У сознания же объяснения пока что нет. Есть понимание, что материя первична, что это материя мозга, но так чтобы полностью объяснить, в идеале воспроизвести появления сознания пока что не получается. В отличии от воспроизведения первого РНК-организма, кажется в 2011 году у японских учёных это получилось.
Аноним 24/10/22 Пнд 22:51:07 #166 №566745 
>>566732
Чего будет если такую сожрать?
Аноним 24/10/22 Пнд 23:37:01 #167 №566746 
>>566745
Высрешь без изменений. Это же керамика. Но риск рака жопы повысится на сколько-то процентов.
Аноним 25/10/22 Втр 17:41:58 #168 №566782 
Слышали про циклоплан? Как вам? По статьям одни + у него, а минусов не подвезли. Да и оригинальную дипломную работу не найти. Хотелось бы посмотреть на неё.
Аноним 25/10/22 Втр 19:58:53 #169 №566788 
>>566782
> Состав: Клестер из рогов оленя пятнистого, трионикс китайский (черепаха); раковины устриц; кладка яиц богомола, женьшень; астрагал перепончатый; дудник китайский; пион молочноцветковый; сыть округлая; аспарагус кохинхинский; солодка уральская; реманния клейкая; звездчатка вильчатая; шалфей многокорневищный; диоскорея супротивная, ямс китайский; эврила устрашающая.

Никогда ещё мел не называли столькими именами.
Аноним 25/10/22 Втр 20:29:40 #170 №566791 
>>566788
Я подумал он про какой-то круговой летательный аппарат написал, лол
Аноним 25/10/22 Втр 23:48:29 #171 №566801 
download.jpg
ХЗ где спрашивать, спрошу здесь.
В США публикуется подробная криминальная статистика, есть куча нон-профитов и СМИ, её обрабатывающих.
Так вот, 50% убийств в США остаются нераскрытыми, а 95% насильников (по тем же параметрам, что в РФ подпадают под статьи "изнасилование" и "насильственные действия сексуального характера") не получают наказания. Это такие официальные данные на основе ФБР и Минюста.
С наказанием преступников по нарушениям вроде краж, грабежей, нападений, вандализма скорее всего дела обстоят даже хуже (т.е. 99% не наказываются).
В связи этим получается, что полицейский аппарат и тюрьмы это театр безопасности в чистом виде. В лучшем (лучшем!) случае это аппарат гос. террора для запугивания склонной к преступности части населения.
Я не прав? Ни разу не видел подобной критики полиции, я придумал хуйню же, да? Но статистика верная, никто с ней не спорит. Подавляющее большинство преступлений остаются без наказаний, в том числе самых тяжких, даже в полицейских (и богатейших) США. как так?
Аноним 26/10/22 Срд 00:02:40 #172 №566802 
>>566801
Что-то ОЧЕНЬ сомнительно.
Обычно преступления совершают не очень умные люди. А это отпечатки, днк, камеры плюс много где, телефоны пеленгуются постоянно, все с ними ходят и не думают не брать в каком-то случае, машины отлсеживаются все куда какие ездят маршруты.
Посмотри сериал типа Декстера, или подумай хотя бы головой, мысленный эксперимент проведи, попробуй мысленно совершить преступление и с высокой вероятностью остаться не пойманным.
Это сложно и нужно быть реально умным чтобы С ВЫСОКОЙ ВЕРОЯТНОСТЬЮ (а не случайно) не попасться.
Аноним 26/10/22 Срд 00:10:39 #173 №566803 
>>566802
>Посмотри сериал
Кстати, гуглил этот вопрос по ключевым словам на английском, один чел в рилейтед треде написал, что его батя из убойного отдела говорил, что Голливуд насколько "большой" потому, что население должно верить, что копы обязательно всех покарают.
Повторюсь, сейчас в США 50% убийств не раскрываются. Это в заголовках мэйнстримной прессы. Даже не убийств, а смертей, официально зарегистрированных как убийства, т.е. на деле доле ещё больше.
Аноним 26/10/22 Срд 00:14:09 #174 №566804 
>>566803
Я не говорю что это прям точно неправдивая статистика, их банально может просто не ловят ленивые копы. Я только сказал что ловить можно почти всех.

>Повторюсь, сейчас в США 50% убийств не раскрываются.
Мало ли что ты сказал, ты не знаешь насколько достоверна статистика. Может так, может нет, но ты не можешь использовать это как неосопримый аргумент.
Аноним 26/10/22 Срд 00:18:36 #175 №566805 
>>566803
>сейчас в США 50% убийств не раскрываются.
Скорее всего это не преступления как ты их себе представляешь, а разборки между бандами. А правительство и банды в законе и бизнесс сам понимаешь как связаны, это фактически переплетённые штуки.

И кстати ты фигню какую-то себе выдумал, вот сейчас когда писал про разборки банд, так 95% фильмов и сериалов про это, и никто там никого не ловит, иначе бы все сериалы заканчивались бы на первой серии.
Очень мало сериалов и фильмов таких как ты имеешь ввиду, а ты нафантазировал себе что почти все сериалы и фильмы исключительно про то как ХОРОШИЕ КОПЫ ВСЕХ ПОЙМАЛИ И НАКАЗАЛИ.
Аноним 26/10/22 Срд 00:19:44 #176 №566806 
>>566804
>ты не знаешь насколько достоверна статистика
Мне тогда требуется признать, что ФБР или полиция публикуют ложную статистику по убийствам, завышая их количество. Это никак не опровергает вывод про бесполезность, ни посылку про то, что преступников не наказывают.
Аноним 26/10/22 Срд 00:22:51 #177 №566807 
>>566805
>почти все сериалы и фильмы исключительно про то как ХОРОШИЕ КОПЫ ВСЕХ ПОЙМАЛИ И НАКАЗАЛИ
Все фильмы и сериалы - про то, что криминальные (т.е. не фантастические борцы с преступностью и прочие мстюны) убийцы либо погибают, либо оказываются в тюрьме. Буквально все. Вне зависимости от того, какой хороший и харизматичный протагосинт-убийца. В финале он всё равно умрёт.
Аноним 26/10/22 Срд 00:26:58 #178 №566808 
>>566807
>убийцы либо погибают, либо оказываются в тюрьме. Буквально все.
Нет там такого, за сериал сотни трупов, а копы кого-то ловят пару раз.
Ну и кстати первое что ты пишешь только подтверждается, было 2 убийца, один убийца убил второго. Итого 2 трупа, а словить можно 0 преступников.
Аноним 26/10/22 Срд 00:30:20 #179 №566809 
>>566808
>Нет там такого, за сериал сотни трупов, а копы кого-то ловят пару раз.
Ну так если копы поймали, то бог наказал. Главное, что наказание неотвратимо. Знакомая фраза. Это так на экране. В реальности нет. Не для каких-то "банд", а для бытовых преступлений. 95% насильников не в тюрьме, например. Сложно расследовать и доказать, жертвы не сообщают - и всё. И это в США.
Аноним 26/10/22 Срд 00:31:13 #180 №566810 
>>566809
>если копы не поймали, то бог наказал
Аноним 26/10/22 Срд 00:39:04 #181 №566811 
>>566810
>если копы не поймали, то бог наказал
Так это, ещё раз говорю, наоборот подтверждает адекватность статистики и нормальность происходящего, дурик.

Да и как это должно по-твоему происходить когда банды конкурируют? Естественно убивают друг-друга бандиты много.
Аноним 26/10/22 Срд 00:42:46 #182 №566812 
>>566809
>жертвы не сообщают
Ты ж говорил это по ЗАРЕГИСТРИРОВАННЫМ нераскрытым преступлениям.

А преступники тупые обычно, отпечатки, генетический материал, другие следы, пеленгация телефонов, камеры, передвижение машин.
Я же говорил тебе что бредовой выглядит статистика если она как ты сказал по ЗАРЕГИСТРИРОВАННЫМ преступлениям.

Возможно это просто фантазии какой-то селёдки-журнашлюхи, которую выебли и бросили а она надумала что её изнасиловали и написала статью типа "девочки такие бедные, их насилуют а 95% не сообщают, на самом деле 95 насильников не наказывают((9"
Аноним 26/10/22 Срд 00:45:42 #183 №566813 
>>566811
Как, блядь, сюжетные тропы в кино могут что-то говрить о статистике?
>банды
Да, значительная часть убийств в США это бандитские разборки в этнических гетто. С твоей точки зрения полиция, суды и прокуратура настолько в доле с ними, что сознательно игнорирует глаза на огромную долю тяжких преступлений в стране. Это как-то опровергает вывод, что полиция нихуя не делает?
Аноним 26/10/22 Срд 00:49:03 #184 №566814 
>>566812
>Ты ж говорил это по ЗАРЕГИСТРИРОВАННЫМ нераскрытым преступлениям.
Про изнасилования не говорил.
Говорил про убийства - регистрируются не все криминальные смерти (это невозможно), так что статистика раскрываемости завышена в любом случае.
>преступники тупые обычно, отпечатки, генетический материал, другие следы, пеленгация телефонов, камеры, передвижение машин
50% убийств в США не раскрываются. Ну, это такая статистика. Она выдумана полицией, ФБР, прокуратурой и т.д.? ОК, они бесполезны или сами преступники. На что ты возражаешь тогда?
Аноним 26/10/22 Срд 00:50:35 #185 №566815 
>>566812
>Возможно это просто фантазии какой-то селёдки-журнашлюхи
Это официальные данные ФБР и Минюста, никто с ними не спорит. В США 95% насильников не получают наказания.
Аноним 26/10/22 Срд 01:04:12 #186 №566816 
>>566813
>С твоей точки зрения полиция, суды и прокуратура настолько в доле с ними, что сознательно игнорирует глаза на огромную долю тяжких преступлений в стране.
Так а смысл копам сильно стараться искать кого-то когда преступник убил другого преступника, и к тому моменту когда его найдут он скорее всего будет уже убит ещё другим преступником?
Просто какой смысл его искать?
А целиком с бандами борятся другие ведомства, и когда накрывают банду, преступления зарегистрированные в другом ведомстве остаются нераскрытыми.
Это не значит что с бандами не борются, это значит что заводят много разных дел.
Аноним 26/10/22 Срд 01:07:31 #187 №566817 
>>566816
>смысл копам сильно стараться искать кого-то когда преступник убил другого преступника
Действительно. Мегамозг в треде.
Аноним 26/10/22 Срд 01:09:13 #188 №566818 
>>566815
>никто с ними не спорит
Читай мелким шрифтом, мб там что-то "экстрапаляции основанной на фантазии дырки о том что 95% женщин вообще не сообщают якобы если их реально изнасиловали"

В любом случае не имеет смысла обсуждать что-то когда у тебя нет точных данных которые ты сам получил или они получены с высокой вероятностью получены в строгом соответствии с научным методом, а не ВБРОС для каких-то политических целей, и не размещённый в виде статьи в газете.
Это всё-таки раздел науки, тебе дали несколько вариантов как и почему это может происходить, более подробное обсуждение не имеет смысла, ведь по факту данных у тебя нет, это просто флуд будет и фантащирование.
Аноним 26/10/22 Срд 01:12:28 #189 №566819 
>>566817
Таков мир и другим не будет.
Про экономическую эффективность слышал?
Можешь сам отчислять каждый месяц 1000$ на работу ментов вместо 20$, и сам во вторую смену подрабатывать, но сомневаюсь что ты будешь это всё делать, так что жри как есть, всё ещё очень хорошо, жаль только что социалистов и коммунистов не отстреливают как собак полицейские.
Аноним 26/10/22 Срд 01:16:15 #190 №566820 
>>566818
>мб
Нет, не мб. Ещё будут фантазии?
>ВБРОС для каких-то политических целей
ФБР годами публикует национальную статистику по убийствам. В том числе с разбивкой по расовой принадлежности преступников и жертв. Негры совершают их чаще. Как и прочие преступления. Весь форчан засран боевыми картиночками, что с учетом разницы в численности негры в 45 чаще чаще нападают на белых, чем наоборот. Это заговор чтобы опорочить негров или тут ФБР внезапно сообщает известные им данные как есть?
Аноним 26/10/22 Срд 01:16:46 #191 №566821 
>>566819
Т.е. ты согласен, что полиция бесполезна, ОК.
Аноним 26/10/22 Срд 01:27:55 #192 №566822 
>>566820
>>566821
Мне насрать на твои фантазии в которых нет научного метода.
Аноним 26/10/22 Срд 01:39:26 #193 №566824 
>>566801
>50% убийств в США остаются нераскрытыми
Почему, там же можно пытать людей нехваткой сна и использовать вырванные из контекста слова как доказательства? В США же постоянно казнят невиновных, что мешает им просто взять и "раскрыть" все убийства?
Аноним 26/10/22 Срд 01:39:37 #194 №566825 
>>566822
>научного метода
Буквально не существует, кстати. Можешь спросить любого научного сотрудника, если не веришь.
Аноним 26/10/22 Срд 01:42:11 #195 №566826 
>>566825
Ты тупой. Научный метод не может не существовать, он чётко сформулирован. То что его кто-то не использует так это другое дело, они просто всеми адекватами выписываются из учёных.
Аноним 26/10/22 Срд 01:42:59 #196 №566827 
>>566824
>что мешает им просто взять и "раскрыть" все убийства
То, что полиция и прочие существуют не для раскрытия преступлений, а для театра безопасности и/или запугивания маргиналов. Я так считаю. Пока что никто не опроверг.
Аноним 26/10/22 Срд 01:44:21 #197 №566828 
>>566826
Чел, просто спроси в интернете какого-нибудь кандидата наук. Биолога или физика. Найди и спроси. ОТВЕТ УДИВИТ!
Аноним 26/10/22 Срд 01:46:08 #198 №566829 
>>566828
Ты очень тупой, прям тупорылый, сорян.
Не можешь отличить "не существует" и "иногда/часто его кое-где/много где не придерживаются".
Аноним 26/10/22 Срд 01:49:03 #199 №566830 
>>566827
>Пока что никто не опроверг.
Никто и не должен опровергать фантазии дауна на научной доске.
Тебе тут максимум что могут просто сказать соответствует что ты высрал научному методу, не соответствует, и может подсказать как это будет правильно по научному методу. Соответственно и все твои вопросы должны строиться в этом ключе, или это просто флуд и ты идёшь нахуй.

Тебе с тем что ты высрал на другую доску, например в по/, но там вроде абу забанил всех свиней.
Аноним 26/10/22 Срд 01:49:20 #200 №566831 
>>566829
Чел. Чел...
Спроси учёного. Со справкой. Раз этот твой метод "чётко сформулирован", то учёный тебе легко ответит, что это такое, да?
Не проверяй, придерживается ли он его, просто спроси.
Я гарантирую, что с вероятностью в 80% он просто чёта промычит. В 20% выдаст общие слова, и не те, что ты себе нафантазировал или вычитал из википедии.
Аноним 26/10/22 Срд 01:51:02 #201 №566832 
>>566831
Ты тупорылый. Бумажка ничего не значит, значит только соответствие информации/исследования/эксперимента/метода получения результатов, метода построении теории научного методу.
Аноним 26/10/22 Срд 01:51:29 #202 №566833 
>>566830
>Посылка 1
>Посылка 2
>Вывод
В душе не ебу, какая тебе наука нужна. Либо опровергай посылки, либо указывай на ошибку в умозаключении.
Аноним 26/10/22 Срд 01:52:29 #203 №566834 
>>566832
Вот, и ты понимаешь, что учёные понятия не имеют о твоём методе.
Аноним 26/10/22 Срд 01:52:45 #204 №566835 
>>566831
Я тебе уже выше написал что такие просто выписываются из учёных согласно научному методу.
Это не значит что научного метода не существует, это значит что те кого ты считаешь учёными ими не являются.
Нормальные люди в отличии от тебя их учёными не считают, просто у тебя iq <80
Аноним 26/10/22 Срд 01:53:54 #205 №566836 
>>566833
У тебя плохо с памятью, я уже сказал что у тебя нет данных полученных в соответствии с научным методом, всё.
Хватит спамить тут.
Аноним 26/10/22 Срд 01:56:54 #206 №566837 
>>566835
>>566836

Заебали срать хуйнёй какой-то!
Аноним 26/10/22 Срд 01:57:16 #207 №566838 
>>566835
>такие просто выписываются из учёных
У тебя и специальная книжечка для этого есть, я угадал? Ну ты рофел каешна.
>>566836
Смерть и причины смерти регистрируются по шаманским методам, статистика собирается и обрабатывается по методам авгуров, я тебя понял.
Аноним 26/10/22 Срд 02:05:05 #208 №566839 
>>566838
Дебилоидик, ты не собирал статистику, у тебя нет статистики, у тебя есть оглашённые данные, оглашение которых с высокой вероятностью преследует какие политические цели. Имеет место личная заинтересованность, что делает эти данные с высокой вероятностью предвзятыми.

В научных исследованиях сообщить какую-то неправильную фундаментальную константу не имеет особого смысла для сообщающего, что делает данные о ней намного более вероятно верными.

Когда сам соберёшь статистику тогда приходи.
Аноним 26/10/22 Срд 02:12:55 #209 №566840 
>>566828
Чувак с "Химии - просто" - научный сотрудник, и он говорил, что научный метод есть. инб4: он уволен неделю назад!!1
Аноним 26/10/22 Срд 02:17:40 #210 №566841 
>>566839
>ты не собирал статистик
ФБР собрало. Ты считаешь, что это заговор такой. Потому что она тебе почему-то не нравится. Ноука!
>>566840
Видео-конент для детей? Да, в нём есть научный метод, я согласен.
Аноним 26/10/22 Срд 02:24:49 #211 №566842 
>>566841
Ты хочешь обсудить научный метод?
Аноним 26/10/22 Срд 03:28:34 #212 №566843 
>>566842
Нет, я хочу чтобы ты принял свои таблетки и съебал.
Аноним 26/10/22 Срд 12:17:55 #213 №566858 
>>566802
>А это отпечатки, днк
Это не убийство лорда в чистеньком поместье, а ночью в темном переулке в гетто шмальнули пару раз, подобрали дропнутый лут и вдарили по тапкам, труп нашли спустя неделю.
>камеры плюс много где
Подозрительная плохо освещенная личность в капюшоне возрастом от 20 до 40 лет свернула за угол и на этом все
>все с ними ходят
Я сомневаюсь что они отслеживаются с точностью до пары метров.
Аноним 26/10/22 Срд 15:31:59 #214 №566862 
>>566705
> Если человек будет дышать смесью в которой кислорода столько же сколько обычно, а все остальное углекислый газ, то как он будет себя чувствовать?
Никак. При достижении концентрации в крови СО2 8% угнетается дыхательный центр. R.I.P.
Аноним 26/10/22 Срд 17:12:06 #215 №566864 
>>566843
Не рвись дурочка тупая.
Аноним 26/10/22 Срд 17:14:00 #216 №566865 
>>566717
Давление не изменяет же концентрацию.
Эффективность работы газообменной мембраны наверное изменится при изменении давления, но прямой зависимости нет.
Аноним 26/10/22 Срд 18:40:56 #217 №566869 
>>566865
>Давление не изменяет же концентрацию.
P = c·R·T;
Pi = P°i·χi
Аноним 26/10/22 Срд 18:42:50 #218 №566870 
>>566865
А, стоп, проебался, в том смысле в котором может влиять на работу мембраны, влияет.
Я чёт про соотношение углекислый/кислород подумал.
Аноним 27/10/22 Чтв 09:16:00 #219 №566891 
image.png
image.png
image.png
Что за ебаный бред.mp4
О чем это видео, какие данные использовали и для кого создано?
https://www.youtube.com/watch?v=MJ50knyZGCE&t=75s
Аноним 27/10/22 Чтв 09:38:10 #220 №566892 
>>566891
Длина и даже количество молекул ДНК отличается в разы, что там может совпадать?
Аноним 27/10/22 Чтв 11:46:57 #221 №566894 
>>566892
Может кодируемые участки?
Аноним 27/10/22 Чтв 12:15:18 #222 №566895 
>>566892
>>566894
Разве не у всех живых организмов должна быть куча общих генов? И чем они родственнее друг друг другу тем их больше.
Аноним 27/10/22 Чтв 13:28:47 #223 №566898 
>>566895
Суть не в генах, а в их вариациях и взаимодействие с друг другом.
Один и тот же ген(конечный белок) у разных организмов может по разному записан в ДНК, это только влияет на его экспрессию.
И ДНК это не только гены, а скопище эволюционного говна, которые не отсеивается отбором лишь потому, что они нейтрально сцепливаются с генами.
Аноним 27/10/22 Чтв 13:47:16 #224 №566899 
>>566895
Половина человеческих генов в этих карточках просто не поместится. Не куча, кучка.
>>566898
Так пусть прямо пишут что белки совпадают, зачем врать про совпадение ДНК?
Аноним 27/10/22 Чтв 15:41:39 #225 №566901 
>>566899
Такие значит наупоперы и журнашлюхи, которые не могут объяснить, что ген и ДНК это разные вещи.
Аноним 28/10/22 Птн 02:06:12 #226 №566908 
Мы стоим на улице, на нас сверху давит воздух, мы заходим в помещение, воздух начинает давить на крышу здания, почему мы не чувствуем никакого облегчения?
Аноним 28/10/22 Птн 02:29:56 #227 №566909 
>>566908
Если здание герметично как подводная лодка то разница будет.
Аноним 28/10/22 Птн 02:31:49 #228 №566910 
>>566909
А что будет с давлением если у здания связь с внешним миром только на нижнем уровне?
Аноним 28/10/22 Птн 03:03:40 #229 №566911 
>>566908
Воздух в помещении уже сжат, разжаться он не может, потому что в дырдочки в дверях/окнах его не пропустит внешнее давление. Поэтому возможен только обмен воздушными массами, а не отток атомов из локального зоны. Аналогично, если ты залезешь в бочку и закроешь герметичную крышку - воздух так и останется сжатым.
Понизить давление можно лишь принудительно, мощным насосом или вентилятором, как делают для проверки теплоизоляции помещений.
> если у здания связь с внешним миром только на нижнем уровне?
Внешнее давление вдавит воздух в верхние уровни, снизу вверх. После герметизации здания воздух останется сжатым.
Аноним 28/10/22 Птн 08:00:59 #230 №566914 
>>566908
Нет никакого столба, это абстрактная выдумка для школьников
Аноним 28/10/22 Птн 08:03:19 #231 №566915 
>>566910
Если хочешь почувствовать что-то, то тебе надо залезть на высокую гору, 5 километров сойдет. Люди описывали состояние на вершине близкое к оборочному, теряли равновесие даже от движений глазами
Аноним 28/10/22 Птн 08:07:22 #232 №566916 
image.png
image.png
image.png
image.png
Есть правдоподобная версия происхождения таких челюстей?
Аноним 28/10/22 Птн 09:33:59 #233 №566917 
>>566916
Происхождение от пиявок, толстые шкуры, пила как способ добычи крови.
Аноним 28/10/22 Птн 11:17:53 #234 №566924 
>>566916
Осьминог выеб акулу.
sage[mailto:sage] Аноним 28/10/22 Птн 20:21:22 #235 №566941 
16624378978020.png
Насколько хорошо видит человек с 100% зрением? Насколько далеко?, в какой мере?
sage[mailto:sage] Аноним 28/10/22 Птн 20:38:41 #236 №566942 
16624378978020.png
Забыл еще спросить позволяет ли лазер корекция востановить зрение до почти идеального? или эт только влажные фантазии?
Аноним 28/10/22 Птн 22:31:44 #237 №566948 
IMG20210329170253.jpg
IMG20210309142626.jpg
Здравствуйте. Посоветуйте пожалуйста яркий и фундаментальный учебник по этологии.

Типа как в своих областях: Фейнмановские лекции по физике в 9 томах Ричарда Фейнмана☘️ (он крут как Грета Тунберг 💚) или Биология в 3-х томах Тейлора Грина Стаута. Я их прочитал в свое время just for fun и личного развития и они мне очень понравились.

P.S. Никаких русских авторов.
Аноним 29/10/22 Суб 00:34:23 #238 №566953 
>>566941
Я вижу как минимум на 20 миллионов световых лет, т.к. вижу ядро галлактики Андромеда невооружёным глазом.

>>566942
Да, позволяет, жене отфемтосекундили оба глаза, было -6, теперь идеальное, периодически проверяю ещё по Мицару\Алькору и плеядам, а так же всяким проводам-антенкам на удалённых зданиях. Прошло уже лет 10 со времени операции, зрение до сих пор идеальное, хотя было уже 2 родов(инбифо съеби на пикабу, но я поясняю надёжность и долговечность результата даже при ебических нагрузках).
Аноним 29/10/22 Суб 00:49:26 #239 №566954 
image.png
Подскажите форму для игрушечного снаряда(делаю игрушечный хаймарс, но чтоб пулял), запускаемый обычной пружиной сжатия из трубки. Чтобы летел на 15-20 метров не вихляя. Пикрил не подходит, снаряд(деревянный нагель 16мм) летит вообще хуй пойми куда, кувыркаясь. Хотя имеет стабилизаторы из пластиковой бутылки, которые не только раскрываются в полёте, но и имеют спиральную(винтообразную) форму.
Аноним 29/10/22 Суб 01:01:38 #240 №566955 
>>566942
Я сделал лазерную коррекцию больше 20 лет назад. Это хаймарс!

После всей юности полуслепой, даже сегодня иногда останавливаюсь и охреневаю от HD детализации окружающего мира. Десятки лет а я не привык до конца насколько мир красив.
Ну и как не охеневать, если после того, как рос с тумане войнышкольной я теперь вижу солнечный блеск на леске удочки за 500 метров.
Аноним 29/10/22 Суб 03:51:02 #241 №566957 
>>566941
>>566942
Говорят, с возрастом развивающаяся дальнозоркость может компенсировать близорукость. Как-то слабо верится, зато бесплатно.
Аноним 29/10/22 Суб 12:07:41 #242 №566963 
Скажите как ученые получают антиматерию, на какой установке?
Аноним 29/10/22 Суб 12:36:20 #243 №566965 
>>566963
коллайдер
Аноним 29/10/22 Суб 22:06:04 #244 №566990 
>>566965
Дома можно какую приспособу для этого смастерить?
Аноним 29/10/22 Суб 22:24:20 #245 №566991 
>>566990
>можно
Да. Юридического запрета нет.
Аноним 29/10/22 Суб 22:56:15 #246 №566994 
>>566990
Есть даже ускорители частиц на чипе.
Аноним 30/10/22 Вск 09:29:31 #247 №567003 
>>566991
Учитывая сколько энергии она жрет, нужно рядышком АЭС организовать
Аноним 30/10/22 Вск 15:01:33 #248 №567018 
>>566994
Херасе, они еще и в 10 раз эффективнее линейных ускорителей, если, конечно, ученый не изнасиловал журналиста. До чего дошел прогресс, потомки будут майнить и передавать по интернету какие-нибудь позитроны, лол. А у нас вместо этого создатут очередные аналоговнет модификации калаша и Т-72, мдя...
Аноним 31/10/22 Пнд 15:20:27 #249 №567048 
>>566650
Ну, а если таки инвазивная стимуляция зон мозга? С крысами вроде работает.
Аноним 31/10/22 Пнд 20:44:17 #250 №567083 
1667238257004.png
Как посчитать давление, если перед сжатием оно было атмосферным?
Аноним 31/10/22 Пнд 23:16:36 #251 №567088 
>>567083
Тепло уходит или не уходит?
Аноним 01/11/22 Втр 00:40:34 #252 №567092 
>>567088
Нет, всеми потерями и трениями можно пренебречь.
Аноним 01/11/22 Втр 00:41:34 #253 №567093 
>>567092
В 4 раза больше чем начальное давление.
Аноним 01/11/22 Втр 00:59:10 #254 №567094 
>>567093
Но я не это спросил же...
Аноним 01/11/22 Втр 01:07:23 #255 №567095 
>>567094
Ну а сам прикинуть если ответ дан?
В 2 раза даст уменьшение объёма в 2 раза
И 2 раза даст увеличение температуры в 2 раза, из-за уменьшения объёма в 2 раза.

Или по-другому ответ-считается из уравнения состояние идеального газа.
Аноним 01/11/22 Втр 01:16:23 #256 №567096 
1667254583217.png
>>567095
А если так? Насколько объем изменился мы не знаем, зато знаем что поршень сжали с силой 100кг Х 10кгм/с2 = 1кН и внутри находился воздух с атмосферным давлением и стандартной температурой. Рано или поздно поршень остановится и где он остановится зависит от силы и диаметра, сила и диаметр известны. Как найти конечное давление?
Аноним 01/11/22 Втр 01:17:26 #257 №567097 
image.png
>>567092
Уравнение аддиабаты. Бери подставляй
Аноним 01/11/22 Втр 01:20:38 #258 №567098 
>>567096
В первом случае ты тоже условно знаешь силу
Давление в 4 раза, значит больше начального в 4 раза, давленик умножить на площадь равно сила
Точно так же выделяется из этого во сколько сжалось.
Аноним 01/11/22 Втр 01:23:34 #259 №567099 
>>567097
>>567098
Короче буду бамповать этот >>567096 вопрос пока не дадите формулу, потому что я не понимаю ваших ребусов с адиабатами и прочим.
Аноним 01/11/22 Втр 01:24:49 #260 №567100 
Мне этот чувак напомнил то что я хотел спросить.

Какого хера когда берёшь пакет, его конец в пучок рукой сдавливаешь, чтобы там внутри воздух был закрыт, но как бы не натянут, с запасом плёнки пакета для того объема было

И слегка бить таким пакетом по столу. Пока запас плёнки есть, пакет совсем без усилий изменяет форму в момент удара по столу. А замет рука чувствует резкий удар.
Но это ведь газ, он должен идеально равномерно и гладенько сжиматься, с постепенным наростонием усилия.
Попробуйте, не могу понять хули оно так.

Берёшь пакет маленький из под булочек, сжимаешь его открытый конец рукой чтобы там внутри пузырь воздуха был, но свободный, не сжатый, и хлопаешь легонько таким пакетом по столу.
Воздух ведёт себя как несжимаемая штука.
Аноним 01/11/22 Втр 01:26:58 #261 №567101 
>>567099
PV=XT
P-давление
V-объём
Х-количество газа
Т-температура в кельвинах
Аноним 01/11/22 Втр 01:34:08 #262 №567102 
>>567101
Я не знаю объем, так что мимо.
Аноним 01/11/22 Втр 01:42:06 #263 №567103 
>>567102
>>567099
Тебе дали формулу, если не можешь подставить, то ты кретина кусок, тебе чисто из вредности никто не будет помогать, чтобы не кормить идиота.
Аноним 01/11/22 Втр 01:44:15 #264 №567104 
>>567102
Дебил, ты знаешь объём.
Аноним 01/11/22 Втр 01:45:22 #265 №567105 
>>567101
Бля, я это сам написал, но
Получается идеальный газ как бы одновременно и имеет теплоёмкость, но при этом и имеет нулевую теплоёмкость?
Т.е. его темплоёмкость не зависит от количества судя из формулы.
Аноним 01/11/22 Втр 01:46:20 #266 №567106 
>>567105
Ты совсем конч? Теплоемкость идеального газа на один моль указана
Аноним 01/11/22 Втр 01:46:52 #267 №567107 
1667256410462.jpg
1667256410474.jpg
>>567103
>>567104
Типичные пидорахи. Дали ответ, неправильный, уже час высираются на форуме доказывая обратное, хотя правильный ответ можно получить в первой ссылке гугда за две секунды если знаешь английский язык. Омерзительно. Вот ответ анончик >>567096
Аноним 01/11/22 Втр 01:48:58 #268 №567108 
>>567106
Ну так посмотри формулу
Она подразумевает, что добавив х2 количества газа, давление тоже вырастет в 4 раза, т.к. температура вырастет в 2 раза ещё.
Но какой температуры будет добавленный газ?
Условия этого нет, получается он увеличит температуру ровно как газ который был, т.е. своей теплоёмкостью не повлияет на процесс.
Аноним 01/11/22 Втр 01:49:37 #269 №567109 
>>567107
Дебил, зачем ты сам себе отвечаешь?
Аноним 01/11/22 Втр 02:14:45 #270 №567112 
>>567108
Ты не понимаешь как работает идеальный газ вот вообще, читай учебники иди
>>567107
Это вообще к другой задаче ответ, ты либо свою не мог сформулировать, либо совсем маленький и не понимаешь, что от тебя спрашивают.

P.S. Я понять не могу, а какого хуя тут вопросы уровня седьмого класса? Почему школота не идет в \un?
Аноним 01/11/22 Втр 03:07:52 #271 №567114 
>>567112
>Ты не понимаешь как работает идеальный газ вот вообще, читай учебники иди
Конкретно по пунктам распиши ошибки в том что я написал, если не балабол и знаешь.
Аноним 01/11/22 Втр 11:57:56 #272 №567125 
>>567048
Инвазивная стимуляция до сих пор это просто слабый удар дубиной, который взывает стрессовую реакцию. В силу пластичности мозг адаптируется на этой раздражитель. Это по определению калечащее действие, ибо угнетает функции здоровые функции.
Аноним 01/11/22 Втр 18:24:46 #273 №567149 
изображение.png
Почему при варке пелемени сначала тонут, а потом всплывают?
В них же лёд, а при таянии льда плотность только увеличивается.
Аноним 01/11/22 Втр 18:51:18 #274 №567151 
>>567149
В тесте есть крахмал и белок(клейковина). В сыром виде они плотно связаны с друг другом в плотную массу. При варке крахмал гидролизуется - его ветвистая структура разрушается на отдельные фрагменты, помимо этого разрушается белок, который связывается с отдельными фрагментами крахмала. Так из плотной структуры получается очень рыхлая аморфная структура, у который плотность ниже исходной.
Если тесто запихнуть в место горячее, процесс пойдет дальше и плотность еще сильнее упадет. Так получится хлеб.
Аноним 01/11/22 Втр 18:55:17 #275 №567152 
изображение.png
>>567151
Но макарошки-то не всплывают.
Аноним 01/11/22 Втр 18:59:49 #276 №567153 
>>567152
В тесте для пельменей есть вода, именно это вода и обеспечивает гидролиз крахмала.
Макароны лишены воды и гидролиз крахмала идет позже разрушения клейковины.
Аноним 01/11/22 Втр 19:03:42 #277 №567154 
>>567153
Хмм, может быть.
Не очень помню, как ведет себя свежая, не сушеная лапша. Кажется она всплывает.

Но это наверное не все. Потому что замороженная курочка тоже всплывает при варке.
Аноним 01/11/22 Втр 19:36:39 #278 №567155 
>>567149
Там внутри дохуя воздуха, скорее всего просто от температуры раздувает.
Это у воды плотность от температуры очень слабо меняется, а горячий воздух намного более плывуч в воде чем холодный.
Аноним 01/11/22 Втр 20:43:01 #279 №567157 
Сап. Объясните про спинор, пожалуйста.

1. Спинор это комплексный вектор в 3д пространстве?
2. Почему тогда спиноры и матрицы паули 2д?
3. Что проецируют матрицы паули и куда? Как я понял, они скорее отражают вектор относительно оси симметрии, а не проецируют его куда-то? Но все говорят про какие-то проекции и компоненты по оси, я что-то вообще не понял, откуда бы им там взяться.
Аноним 01/11/22 Втр 21:34:52 #280 №567159 
>>567157
Следи за руками.
- Берем н-мерное комплексное пространство векторов.
- Определяем в нем умножения векторов на друг друга через введение внешнего и внутреннего произведения. Получаем скаляры и бивекторы, три-векторы и прочие н-векторы. Возможно определить произведение во всей совокупность н-векторов породив алгебру Клиффорда.
- Алгебра Клиффорда изоморфна алгебре перемножением квадратных матриц. Это значит каждому скаляру, вектору или н-вектора, а так же их общей совокупности вроде скаляр+вектор, можно сопоставить квадратную матрицу и наоборот.
- Поскольку матрица представляет некоторый линейный оператор, то есть пространство вектором на котором этот оператор действует. Вектор из этого пространство называют спинором.
Спросишь зачем нужное это громоздкое опредление? Так в том, что есть занятная магия вращениями(а так проективными преобразованиями) между изначальным пространством и пространством спиноров. Так группа вращений может быть размазана на линейное пространство и наоборот, что дает мощный алгебраический инструмент.

Теперь к твоим вопросам.
1. Нет, спинор это комплексный вектор в абстрактном пространстве.
2. Поворот в 3д можно представить как некоторый вектор, этот вектору можно сопоставить некоторую комплексную матрицу, которая раскалывается как совокупность матриц паули. Эти матрицы уже действуют на спиноры.
3. Матрицы паули преобразуют спинор в другой спинор, они действуют уже в абстрактном пространстве, которое уже лишено геометрического смысла для первого пространство. Вся суть в связи групп вращений и групп линейных преобразований разных пространств.

Главная магия спиноров открывается, когда мы берем группу вращений в псевдоевклидовом пространстве (вращения X бусты) тут уже открываются новые геометрические смыслы.
Аноним 01/11/22 Втр 21:53:27 #281 №567161 
>>567159
Ну теперь все понятно.
Ну а вообще, я просто пытаюсь понять, как более-менее просто геометрически представить спиноры и матрицы паули на сфере Блоха, без лишней строгости. Есть какая-нибудь диаграмма для даунов?

https://physics.stackexchange.com/questions/204090/understanding-the-bloch-sphere

Вот тут нечто похожее на то что я ищу, пытаюсь разобраться, но что-то не очень идет пока
Аноним 01/11/22 Втр 22:19:06 #282 №567163 
>>567161
Забудь об этом. Спиноры лишены геомеческого смысла в 3д. Весь смысл только алгебраический и то только для пространства-времени.
Сфера Блоха вообще про другое. Это отображение комплексной проективной прямой на вещественную сферу.
Если хочешь выкинуть в геометрию спиноров, то начни изучать повороты в 3д через кватернионы, а потом алгебру Клиффорда.
Аноним 01/11/22 Втр 22:46:31 #283 №567164 
166733192820390733.jpg
>>566163 (OP)
Помогит решить задачу
Аноним 01/11/22 Втр 23:16:44 #284 №567173 
1667333799891.jpg
Задача свободного падения с сопротивлением воздуха, лекция на ютубе. Почему он тут прибавляет скорость на массу (м/с.кг) к ускорению (м/с2)? Что за бред?
Аноним 02/11/22 Срд 01:06:54 #285 №567177 
>>567173
У этой хуйнюшки справа (похожей на r) размерность дурацкая, причем как раз такая, чтобы ускорение выходило. Это типа коэфф, связывающий сопротивление воздуха и скорость.
Аноним 02/11/22 Срд 01:14:48 #286 №567178 
>>567177
Это же безразмерная залупа или ты точно уверен?
Аноним 02/11/22 Срд 07:54:26 #287 №567183 
>>566163 (OP)
Почему отказались от понятия "эфир"? Ведь в эксперименте опровергли лишь одно свойство этой среди, однако современная физика так или иначе подразумевает некую базовую среду - пространство-время, физический вакуум и т.п. По этой среде перемещаются электромагнитные и гравитационные волны, она может искажаться под действием массы вещества и прочее. Так почему бы не оставить старое название?
Аноним 02/11/22 Срд 09:55:22 #288 №567191 
>>567183
Эфир же слишком механистическая модель, в том смысле в нем нельзя делать обобщения движения. Всякие лагранжианы выглядит как уродливое нагромождение.
Если ты попытаешь для этого как-то пригладить эфир, то у тебя получатся кривая модель пространства-времени, где придется отказывается от механической модели, а это сродни отвергания самого эфира.
Аноним 03/11/22 Чтв 00:59:39 #289 №567206 
Бля, двач
Короче, ружьё.
Его можно держать в руках, и тогда будет классически закон "каждое действие имеет равное обратное противодействие", пуля полетит в одну сторону быстро, а ружьё относительно медленно в другую сторону(но не далеко, руки погасят энергию, но прям самая начальная скорость будет как в вакууме если бы разлетались в разные скорости)
А можно ружьё закрепить почти идеально, тогда скорость его обратного отлёта будет меньше намного.

И должен сохраняться закон сохранения импульс. Допустим замедлили на 20м/с скорость отлетающего назад ружья, значит добавили 20м/с к скорости вылетающей вперёд пули.

Но... Очевидно что энергия выстрела одинаковая в обоих случаях.

НО. Если посчитать суммарную энергию ружья и пули в первом случае,
когда оно отлетало назад допустим 25м/с и весило 4кг, а пуля вперёд 400м/с и весила 10гр
И во втором случае когда оно отлетало назад 5м/с и весило также 4кг, а пуля вперёд 420м/с и весила так же 10гр
ПОЛУЧАТСЯ РАЗНЫЕ ЭНЕРГИИ.
Закон сохранения энергии получается не выполняется?
Аноним 03/11/22 Чтв 01:39:18 #290 №567207 
>>567206
Мамка твоя не выполняется. А пуля в обоих случаях получает одинаковую скорость, потому что скорость пули зависит от пороховых газов, а они одинаковые. Остальная энергия если закрепить жёстко уходит в деформацию - тепловую энергию.
Аноним 03/11/22 Чтв 02:00:34 #291 №567209 
>>567191
Ну понятно что не влоб продолжать следовать старой модели, я скорее про само название. Современная физика утверждает что эфира как всепроникающей среды не существует и что э/м волны распространяются как бы сами по себе. Но ведь это пиздежь - волны распространяются как раз в среде пространства-времени, тоже всепроникающей среде но с другим названием и свойствами.
Аноним 03/11/22 Чтв 02:13:46 #292 №567210 
>>567207
Ты туповат.
Аноним 03/11/22 Чтв 02:19:12 #293 №567211 
>>567209
>но с другим названием и свойствами.
Вот поэтому от эфира отшли. Эфир это просто название. Название даёт тот кто сделал теорию.
Эфир была в прицепе об этом, но была слабой теорией и не работала.
Кто-то другой сделал теорию в принципе об этом же, и т.к. он сделал, то он и выбрал ей название, на этот раз не эфир.
И среда это не пространство-время, а совокупность полей, а теория-квантовая теория поля
Аноним 03/11/22 Чтв 04:05:18 #294 №567212 
image.png
Не могу понять третий пункт. Звездочка - это ведь комплексное сопряжение. Но как это может выполняться, если на Z коммутативно?
Аноним 03/11/22 Чтв 09:59:58 #295 №567214 
>>566163 (OP)
Есть тред прикладной химии? Допустим поинтересоваться, где купить щавелевая кислоту как можно дешевле.
Аноним 03/11/22 Чтв 11:48:11 #296 №567216 
>>567209
А среда тогда в чем распространяется? Среда с магическими свойствами не очень хорошая вещь в науки.
Концепция полей и пространства-времени стали бурно развиваться из-за геометричности и минимальности сущности.
Зачем ебаться с неуловимой средой если можно ввести поля и волны как геометрический объект, что очень хорошо работает?
В добавок геометричность позволяет предсказать кучу новых свойств.
Аноним 03/11/22 Чтв 11:54:36 #297 №567217 
>>567212
Каком еще Z?
И вообще считай левые и правые элементы как тензоры с нижними и верхними индексами. В комплексном случае опускание индекса сродни комплексному сопряжению.
Аноним 03/11/22 Чтв 12:14:57 #298 №567219 
>>567209
ЭМ-волна просто какбэ сама себя генерирует. Магнитный компонент генерирует электрический, а электрический - магнитный. Как-то так. Среда в данном случае наоборот вредна, так как вызвала бы затухание волны.
Аноним 03/11/22 Чтв 13:44:22 #299 №567221 
>>567216
>А среда тогда в чем распространяется?
Ни в чем, т.к. она везде.
> Среда с магическими свойствами
Ну почему магическими-то? Все тот же физический вакуум просто описать как среду. А то как-то непонятно выходит - среды вроде и нет, но что-то там искривляется, порождает виртуальные частицы, имеет ограничение по скорости света и т.д.
>>567219
>ЭМ-волна просто какбэ сама себя генерирует
Любая волна сама себя генерирует - потенциальная энергия переходит в кинетическую циклически. Только вот волна распространяется в чем-то.
>так как вызвала бы затухание волны.
Не обязательно, в идеально упругом теле, например, ничего не затухает.
Аноним 03/11/22 Чтв 14:28:31 #300 №567223 
>>567221
Магические, потому что эфир требует противоречивых свойств, которые не вытекают из общих принципов.
Физический вакуум ну никак нельзя описать как среду, в первую очередь из-за того, что он совокупность нескольких фундаментальных полей, связанных общим квантованием.
О виртуальных частицах можно говорит только в рамках теории возмущений, которая по своей сути только инструмент приближенного вычисления.
Ограничение скорости прямое следствие геометрии полей и пространства-времени.
Аноним 04/11/22 Птн 11:16:19 #301 №567253 
Улитки ведь от соли дохнут? Получается, если на нее поссать, она сдохнет?
Аноним 04/11/22 Птн 16:49:09 #302 №567282 
>>567221
> Любая волна сама себя генерирует - потенциальная энергия переходит в кинетическую циклически.
Нет. У механических волн происходит рассеяние энергии в среде, так как эти циклические переходы выполняются не волной как таковой, а частицами среды. ЭМ-волна же генерирует себя вечно именно из-за отсутствия специальной среды.
> Не обязательно, в идеально упругом теле, например, ничего не затухает.
Ага, а идеально жестким прутом можно передавать сообщения на сверхсветовой скорости, шах и мат, Эйнштейн.
Аноним 04/11/22 Птн 21:51:31 #303 №567290 
0NOVnHvD8KCoWuK9tWam5RC2aUUa9m9rAjY7R-DPuT4CjhJvIncEwZx124op0gLo8iw50WxWk5wTfeAnwTySRHv.jpg
Может кто-то подсказать как теоретически найти длину красного отрезка ?
Аноним 05/11/22 Суб 02:35:59 #304 №567296 
image.png
>>567290
Аноним 05/11/22 Суб 09:22:12 #305 №567301 
>>567296
У тебя радиус три, как половина радиуса может быть 2?
Аноним 05/11/22 Суб 11:42:13 #306 №567303 
>>567301
Почему там арктангенс вместо тангенса?
Аноним 05/11/22 Суб 12:11:13 #307 №567304 
>>567303
Т.е. ≈1.24
Аноним 05/11/22 Суб 13:00:24 #308 №567306 
>>567303
Потому что я ебанутый. Да, там тангенс.
Аноним 05/11/22 Суб 13:10:49 #309 №567307 
>>567209
> Ну понятно что не влоб продолжать следовать старой модели, я скорее про само название. Современная физика утверждает что эфира как всепроникающей среды не существует и что э/м волны распространяются как бы сами по себе. Но ведь это пиздежь - волны распространяются как раз в среде пространства-времени, тоже всепроникающей среде но с другим названием и свойствами.
Вот как раз отличие свойств пространства-времени от эфира и приводит к отказу от термина эфир. Свойства слишком сильно отличается. Оно уже не "выглядит как утка крякает как утка, значит это утка". ПВ не имеет тех механистических свойств, ради которых придумали термин эфир. Поэтому логично и назвать его по другому. Свинью не удобно называть уткой, тк утка подразумевает, что она умеет крякать и летать. А свинья не умеет ни того, ни другого, поэтому называть свинью уткой совсем не удобно.
Аноним 05/11/22 Суб 13:36:11 #310 №567309 
>>567307
Если утки не существует, то почему бы ею свинью и не назвать. Эфир как всепроникающая среда не был опровергнут, просто оказалось что у него свойства слишком иные. Такое бывает в истории науки.
Аноним 05/11/22 Суб 13:38:32 #311 №567310 
>>567309
> Если утки не существует, то почему бы ею свинью и не назвать. Эфир как всепроникающая среда не был опровергнут, просто оказалось что у него свойства слишком иные. Такое бывает в истории науки.
Потому что изначально эфир позиционировался как нечто похожее на что то типа газа или жидкости (та самая утка), а по факту позже оказалось, что от газа и жидкости там вообще ничего не осталось (те это на утка даже рядом).
Аноним 06/11/22 Вск 08:45:17 #312 №567337 
Если мы складываем одночлен, приводя в стандартный вид, является ли он в это время многочленом?
Аноним 06/11/22 Вск 09:33:55 #313 №567338 
8d5bd404d3b9438b9757d33b0530.png
>>567337
математика круто
Аноним 06/11/22 Вск 10:23:38 #314 №567339 
Почему медь имея 1 валентиный электрон не является щелочным металлом?
Аноним 06/11/22 Вск 10:25:49 #315 №567340 
>>567339
*валентный
Аноним 06/11/22 Вск 12:20:40 #316 №567344 
>>567339
У меди одиннадцать валентных электронов.
Аноним 06/11/22 Вск 15:30:40 #317 №567347 
>>567339
У меди почти заполненные d орбитали, которые ОЧЕНЬ сильно экранируют один бедный s электрон. Это как куча бабок и тёток с гиперопекой вокруг двощера. В итоге меди проще взаимодействовать с парой d электронов, чем связаться с s электроном.
Другой момент, связывание s электрона возмущает d орбитали так, что у них становится около релятивиский угловой момент. А это уже порождает щель для энергетического спектра металлической связи, что делает медь относительно инертным металлом, а ещё придает характерный цвет.
Аноним 06/11/22 Вск 18:33:17 #318 №567359 
Я не понимаю, а почему из пушки не запускают бпла? Это же очень простой способ запуска, очень дешевый. Дальше можно выпустить крылья и планировать. Че так не делают?
Аноним 06/11/22 Вск 18:38:39 #319 №567360 
изображение2022-11-06183823559.png
>>567359
Что значит "не делают"? Гугли "корректируемый артилерийский боеприпас". Тот же "Краснополь".
Аноним 06/11/22 Вск 19:53:58 #320 №567366 
>>567360
Это не то. Почему нету прямо бпла, но с запуском из пушки? Не обязательно это военная сфера.
Аноним 06/11/22 Вск 21:56:22 #321 №567368 
>>567339
Потому что у меди не один валентный электрон а больше. Медь можно и в +3 сделать.
Аноним 07/11/22 Пнд 11:07:38 #322 №567378 
>>567366
Потому что Ланцет-3 и так эффективен без всяких пушек.
Аноним 07/11/22 Пнд 11:39:06 #323 №567380 
>>567359
А зачем? Надо делать его более прочным, что бы он выдержал ускорение в стволе.
Аноним 07/11/22 Пнд 16:29:21 #324 №567382 
>>567125
Ну ебать, ну должен же быть способ делать себе пиздато и чтобы нихуя за это не было кроме дрочки
Аноним 07/11/22 Пнд 16:37:11 #325 №567383 
>>567382
Мозг подчиняется только сознанию, без контроля над сознанием халявы не будет.
Аноним 07/11/22 Пнд 16:40:52 #326 №567384 
>>567383
Чьим контролем?
Аноним 07/11/22 Пнд 17:09:31 #327 №567385 
>>567384
Какая разница? Ты либо контролируешь содержимое своего сознания, либо нет.
Аноним 07/11/22 Пнд 17:14:24 #328 №567386 
1. В электрон-вырожденом состоянии материи нет электронных оболочек?
2. Возможно ли при сверхвысоком давление частичное разрушение электроного облака? Ну например возьмём свинец. Закинули его под давлеие в x ГПа. И разрушалась только внешняя часть 7s, 7p, 6d и 4f оболочки. А оставшиеся внизу остались в целости или деформировались. Или если начала разрушаться электронная оболочка то сразу и вся.
3. Материя белых карликов это очень плотный газ, похожий на жидкость или все таки твердая материя. Ну типа, если такая характеристика как пррчность материи белых карликов.
3а. Если все таки у белых карликов материя жидко-газовая, то через миллиарды лет, когда белые карлики превратятся в черные, то материя черных карликов будет уже твердым телом или нет?
Аноним 07/11/22 Пнд 18:09:53 #329 №567387 
А как так получается, что частицы в ускорителе частиц в своей системе отсчета длиннее ускорителя во много раз, но там спокойно летают? Че за дела?
Аноним 07/11/22 Пнд 18:15:50 #330 №567388 
>>567387
Так ещё в чём прикол, ускоритель это условно круг, а уменьшение длины происходит перпендикулярно касательной. Получается они со своей точки зрения летают по очень маленькому кружку.
Аноним 07/11/22 Пнд 22:33:02 #331 №567389 
>>567386
Говорю косноязычно, бей бейте меня, лучше обоссыте.
Электронные оболочки это больше условность приближения, когда мы пытается многоэлектронную систему упростить через одноэлектронные решения. Поскольку есть эффект экранирование заряда ядра, то многие уровни можно привязать к конкретному ядру, оставляя только несколько внешних/валентных электронов. Но по честному все электроны размазаны на все ядра и их надо считать валентными.
1. Вырождение это ситуация, когда мы уже не можем упрощать уровни электронов и надо использовать решения для всех электронов.
2. Металлическое состояние при обычном давление и так вырожденное. В куске металлического свинца делокализуются электроны из f и d.
3. Тут полно всякий ньянсов вроде температуры, но для белых карликов просто используется релятивистский горячий электронный газ. В таком случае вырожденная материя представляет из себя сверхпроводящее сверхтекучие твердое тело. Под твердым подразумевается, что вещество представляет из себя расслоение из очень жестких слоев или трубок тока, которые скользят относительно друг друга без трения.
3а. Для учата низких температур нужно учитывать спиновое взаимодействие с ядрами. Тут уже становится важен состав вещества. Вырожденное вещество из гелия-3 и гелия-4 будут радикально различаться. Та же самая сверхпроводимость может пропасть, но вылезут другие безумные магнитные эффекты - гелий-3 станет постоянным супермагнитом, а гелий-4 на границе заимеет проводящий слой.
Аноним 08/11/22 Втр 03:01:09 #332 №567395 
Вопрос из области экономики. Теории игр. ХЗ.
На полке супермаркета стоит набор бутылок пива. Стоимость примерно одинаковая (плюс-минус десяткок-другой рублей, одна ценовая категория), содержимое одинаковое (лагер), производители и лейблы разные (есть привезенное из регионов за тысячи км, есть местные отделения гигантских сетей, есть лицензионные глобальные бренды, есть относительно небольшие местные производители).
Ври этом всё это пиво - говно.
Как так?
Конкуренция не работает? Нет, работает.
В этой стране не могут сварить лагер? Нет, могут, и варят, есть локальные и "федеральные" сорта с нормальным вкусом (их просто нет в супермаркетах). Само говно на полках с технологической т.з. - вполне нормальный пищевой продукт, не брак.
Потребитель не может опознать хорошее пиво? Нет, легко может.
В чём тогда дело?
Аноним 08/11/22 Втр 03:23:58 #333 №567396 
>>567395
>Нет, легко может.
>В чём тогда дело?
Нет не может. Никакая это не теория игр, просто говноеды жрут говно, так зачем напрягаться и что-то менять?
Аноним 08/11/22 Втр 03:35:27 #334 №567397 
>>567395
Я помню году в 2005 перед НГ в Перекрестке была распродажа импортного пива в большом ассортименте, многое стоило даже дешевле расеянского. И люди все равно в основном брали ссанину от балтики.
Аноним 08/11/22 Втр 04:33:28 #335 №567398 
>>567396
Есть опыт, который говорит, что любой потребитель плохого пива сразу распознает например мюнхенские старые бренды как нечто охуенное.
>>567397
Это можно списать на рекламу, бренды не на слуху.
Речь о конкуренции между производителями. Она вполне реальна - например огромные маркетинговые бюджеты мировых брендов не могут совладать с локальными, просто потому, что они локальные, и люди предпочитают их. И это реально локальное (региональное) пиво, не продукция ветви мега-конгломерата. Значит люди делают выбор. Но выбирают всё равно ссанину. Почему?
Аноним 08/11/22 Втр 05:40:06 #336 №567399 
>>567398
> Но выбирают всё равно ссанину. Почему?
Потому что ты дурачек из зажопинска, делающий вывод о том, что пьют люди по ассортименту из ближайшего магнита или пятёрочки, а может даже подвального суточника. Маргиналам пофиг что пить, поэтому пьют самое дешевое. Съезди в центр, там будут разливайки с годным разливным на любой вкус, бутылочный крафт от Эля до стаута, в КБ есть ассортимент импорта типа шпатена. Ты живешь в какой то маргинальной жопе, где хорошее пиво просто ни кому не нужно, поэтому там продаётся только ссанина.
Аноним 08/11/22 Втр 06:08:01 #337 №567400 
>>567399
Пчел, во-первых, шпатен говно. Во-вторых, большинство потребителей пива и чего угодно не могут быть маргиналами по определению.
В-третьих, вопрос не про большинство, а про конкуренцию, т.е. отбор. Даже если вкус и прочая органолептика пива это не первостепенный фактор, это всё равно фактор, людям нравится хорошее пиво. Почему в масс-маркете оно не эволюционировало в хорошее? Разнообразие есть, давление отбора есть, время есть - фенотипического результата отбора нет.
Аноним 08/11/22 Втр 06:15:41 #338 №567401 
>[метафизическое, абстрактное, моральное, обобщающее, вообще любое понятие] это всего лишь набор электрохимия в белковых структурах и химическом супе
Как контрить это с ноучной точки зрения?
Если никак, то почему ноука такая ограниченная и примитивная?
Аноним 08/11/22 Втр 07:03:30 #339 №567402 
>>567398
>Есть опыт, который говорит, что любой потребитель плохого пива сразу распознает например мюнхенские старые бренды как нечто охуенное.
И что с того? Но говноеды продолжают жрать говно.
Если они продолжают, и продажи идут, зачем что-то менять в производстве?
Аноним 08/11/22 Втр 07:05:30 #340 №567403 
>>567401
Ты просто порвался. Слишком слабый и глупый.
Аноним 08/11/22 Втр 12:11:45 #341 №567406 
>>567400
Просто хорошее пиво стоит раза в два-три дороже обычного.
Аноним 08/11/22 Втр 13:13:26 #342 №567408 
Снимок экрана 2022-11-08 в 14.12.23.png
помогите, что не так? последовательность стремится к бесконечности, почему ответ не верный... хз , в том ли треде пишу, но по тематике ничего не найдено. перенаправьте, если не туда пишу
Аноним 08/11/22 Втр 13:33:39 #343 №567409 
>>567406
Это не так, ингредиенты везде одинаковые, техпроцесс тоже. Единственный способ удешевления это использование не ячменного солода, а кукурузы, но такое пиво сразу заметно, это отдельная категория. Остальное это чуть разные сорта хмеля, вода разной жесткости, какие-то мелочи в температурных режимах - ничего, что стоило бы большому производству каких-то отдельных гигантских денег. Это не вино и не коньяк, а всего лишь пиво. Везде стоят современные линии, везде производят продукт формально без дефектов. Но ссанину.
Аноним 08/11/22 Втр 13:43:04 #344 №567410 
>>567409
Однако я на вкус различаю дешевое пиво от дорого. Это при том что пиво я не люблю в принципе и пью его крайне редко.
>Это не вино
Но ведь вино это перебродивший виноградный сок. Шмурдяк Три лопаты за 200 рублей за коробку по идее ни чем не должен отличаться от Шато-де-ля-Мерд за 10 000 долларов бутылка.
Аноним 08/11/22 Втр 13:47:58 #345 №567411 
>>567408
вопрос закрыт, решение найдено
Аноним 08/11/22 Втр 14:02:07 #346 №567412 
>>567410
Сорта винограда сильно различаются (и тупо не рпастут в большинстве стран), можно экономить, добавляя сахара, вкус вина зависит от ферментации и выдержки. Пиво гораздо проще.
Аноним 08/11/22 Втр 21:05:15 #347 №567422 
>>567409
>Это не так, ингредиенты везде одинаковые, техпроцесс тоже.
Как ты тогда объяснишь, что дорогое пиво статистически намного чаще хорошее?
И речь не о ингридиентах, хотя они естественно разные, а о том утверждение что "хорошее пиво дороже" которое ты оспаривать пытаешься.
Аноним 08/11/22 Втр 23:04:09 #348 №567431 
>>567397
>Я помню году в 2005 перед НГ в Перекрестке была распродажа импортного пива в большом ассортименте, многое стоило даже дешевле расеянского. И люди все равно в основном брали ссанину от балтики.
Сейчас я объясню почему это более чем рациональное поведение
Ну во-первых - очевидное ты знаешь что берешь а не кота в мешке.
А во вторых - расскажите сколько ваши выебонистые "крауты" пролежали месяцев на полке (если не лет) пока добрались до вашей кружки, а я поржу.
>распродажа импортного пива в большом ассортименте
>распродажа
Вообще орунах. Никаких подводных.
Аноним 09/11/22 Срд 05:53:51 #349 №567441 
>>567422
Дорогое пиво это или импорт, или крафт (высокая себестоимость из-за малых объемов производства).
>>567431
>сколько ваши выебонистые "крауты" пролежали месяцев на полке (если не лет)
Пиво это буквально алкогольные консервы, если ты не знал, хмель там именно за этим.
Аноним 09/11/22 Срд 11:36:51 #350 №567445 
Говорят, кусок нейтронной звезды размером с горошину весит около сотни миллионов тонн. Если такой кусок отколоть и увезти от звезды он раскукожится до номальных размеров (если оно из железа, то примерно куб 250х250х250м) или останется сжатым?
Аноним 09/11/22 Срд 11:47:58 #351 №567446 
>>567445
Это нестабильное атомное ядро. Если гравитация перестанет его стабилизировать, оно взорвется на хер, а не "раскукожится".
Аноним 09/11/22 Срд 11:53:32 #352 №567447 
>>567446
А если отводить оче медленно?
>>567441
>Пиво это буквально алкогольные консервы
Вкус свежего пива и на исходе срока отличается сильно. Как впрочем и у консерв а еще, если это рыбные, то рыба становится более рыхолй, бееее.
Аноним 09/11/22 Срд 12:31:53 #353 №567448 
>>567447
Оче медленно взорвется нахер.
Аноним 09/11/22 Срд 15:37:33 #354 №567449 
Почему до сих пор не хватает математического аппарата для решения сложных задач? Таких как "Равенство классов P и NP". И когда он появится?
Аноним 09/11/22 Срд 16:42:52 #355 №567451 
>>567449
>когда он появится?
Завтра приходи.
Аноним 09/11/22 Срд 16:44:56 #356 №567452 
>>567451
Хуя ты базаришь чёрт ебаный. Я могу тебя усыновить и забить колышами до полусмерти.
Аноним 09/11/22 Срд 18:03:15 #357 №567455 
>>567445
>Говорят, кусок нейтронной звезды размером с горошину весит около сотни миллионов тонн. Если такой кусок отколоть и увезти от звезды он раскукожится до номальных размеров (если оно из железа, то примерно куб 250х250х250м) или останется сжатым?
А вот интересно. Это нейтроны.
Распадаться они начнут на протоны. Ну и плюс там должны быть электроны, электронейтральность же, все дела.
Получается при распаковке это всё превратится в водород?
Получается, переход через состояние нейтронной звезды превращает все элементы в водород?

А не, стоп... При обратной распаковке же будет обратный синтез происходить, т.е. какой-нибудь уран и золото получатся.
Аноним 09/11/22 Срд 19:03:38 #358 №567457 
>>567445
Смотря откуда откалывать.
На поверхности преимущественно тяжелые ядра.
Чуть глубже коры уже релятивистский горячий вырожденный газ из нейтронов и возмущенных сверхтяжелых ядер. В таком состоянии они удерживаются только гравитации и при снятие давления превращается в тяжелые ядра.
В глубинах твориться магия. Нейтроны там превращаются во всякие гипероны и прочее странное говно. И тут уже вопрос является гипероны в большой куче одним большим ядром и стабильно ли оно при снятие давление. Большинство моделей предсказывают, что таки да, но ебучие старпельки мы не наблюдаем. Тут либо модели надо допиливать или в природе нет естественных процессов вытаскивания их из нейтронной звезды.
Ну и в ядре нейтронной звезды находиться кварковая материя. Тут уже совсем не понято, но большинство моделей предсказывает, что таки да, они стабильны при снятие давления.
Аноним 09/11/22 Срд 19:48:14 #359 №567458 
>>567457
>Тут уже совсем не понято, но большинство моделей предсказывает, что таки да, они стабильны при снятие давления.
Как и то другое не за счёт гравитации а за счёт сильного взаимодействия?
Но мне кажется бред если так, ведь тогда чем крупнее ядро обычных наших земных атомов, в обычных условиях, тем СТРОГО стабильнее бы оно становилось с увеличение.
А то типа всем рулит сильное взаимодействие, стабильность при увеличении уменьшается, уменьшается, уменьшается... А потом хуях, и стабильное на похуях?
И я не про жалкий островок стабильности, а вот про конкретно тупо стабильность полную после определённой черты размера.
Аноним 09/11/22 Срд 22:30:34 #360 №567465 
>>567458
Кварковая материя это тебе не ядро, а отдельное состояние вещества.
Стабильность кварковой материи обеспечивается за счет цветной сверхпроводимости, но для ее возникновения нужен вырожденные газ из кварков. Короче нужна система из громадного количества валентных кварков.
В обычных ядрах валентные кварки все равно по тройкам кучкуются, и чувствуют соседную тройку опосредовано.
Тут можно приветси такую аналогию: нуклон - атом, ядро - молекула, кварковая материя - кристалл металла или крупная молекула с делокализованными связями как бензол.
Аноним 09/11/22 Срд 23:23:30 #361 №567466 
>>567465
Ну это ок, как-то ещё объясняться может, а про то что ты писал до кварковой материи?
Аноним 10/11/22 Чтв 01:31:32 #362 №567468 
>>567466
Страпельки или странная материя?
Упрощенно говоря тут так.
Как известно нуклоны состоят из легких кварков первого поколения(u и d), однако есть кварки второго поколения(тяжелые копии предыдущих) из которого s-кварк(странный кварк) самый легкий.
Аналог нуклона с s кварков называют гипероном, он более тяжелый и быстро распадается на нуклон и всякие мезоны. Гипероны вполне можно как нуклоны в ядро, получив гиперядра или странные ядра.
Однако в квантовой теории есть такой эффект, если нестабильную частицу посадить в достаточно глубокую потенциальную яму, то она станет стабильной. Такую глубокую потенциальную может создать юкавовские(ядерные) силы в достаточно тяжелом ядре. А если брать очень тяжелые ядра, то его "странность" становится выгодна и оно будет стабильнее ядер из нуклонов.

Так же есть и другой более сложный эффект. Его суть в следующим.
Адроны это система кварков, удерживаемые флуктуациями глюонного поля, порождаемые собственным цветным зарядом. Флуктуации настолько сильны, что энергия нужная на их "подавления" составляет значительную долю массы адронов. Когда рядом два адрона, то они помогают друг друга гасить флуктуации сильного поля, но если два адрона подходят совсем уж близко, то они начинают друг друга мешать. Так появляются юкавоские силы, которые позволяют удерживать нуклоны в ядрах вместе, но при этом не позволяют кваркам между нуклонами смешиваться. Когда много нуклонов вместе, то юкавоские силы приходят в насыщение и их интенсивность не растет с увеличением количества нуклонов, что в итоге делает большие ядра не стабильными.
Есть и другой момент, тем тяжелее кварк, тем проще ему подавлять флуктуации глюонного поля. Это снижает интенсивность юкавовский сил, но при этом повышает верхний предел по насыщению сил и позволяет адронам подходит достаточно близко, что порождает новые силы, связанные со смешение кварков между адронами.
Аноним 10/11/22 Чтв 03:07:46 #363 №567470 
>>567468
Бля, ну вот и как тут антропный принцип исключать...
Аноним 10/11/22 Чтв 03:10:54 #364 №567471 
По сути науку уже нужно переделывать в выдумывание всякой странной, на первый взгляд надуманной только ради того чтобы существующее положение вещей могло существовать.
Если так переформатировать науку она возможно станет гораздо более продуктивной.
Чем пытаться адекватные теории выдумывать, которые стараются не множить сущности.
Аноним 10/11/22 Чтв 03:13:22 #365 №567472 
>>567468
На том уровне микромира и плотностей энергий(хотя там уже походу просто слово энергий слишком специальное, больше подойдёт просто "плотностей" без уточнения чего) про которые пишешь, там уже стирается что ли грань между гравитацией и сильным взаимодействием?
Аноним 10/11/22 Чтв 04:46:02 #366 №567473 
Что значит зум e1091? Это в километрах? То есть эта хуйня в примерно 10000000000000000000000000000000000000000000000000000000000000000000000000000000000000000000000000000000000000000000000000000000000000000000000000000000000000000000000000000000000000000000000000000000000000000000000000000000000000000000000000000000000000000000000000000000000000000000000000000000000000000000000000000000000000000000000000 (в один треценсекстригинтиллион) раз больше, чем наблюдаемая Вселенная?

https://www.youtube.com/watch?v=pCpLWbHVNhk
Аноним 10/11/22 Чтв 12:09:57 #367 №567480 
>>567472
Нет.
Сильное и гравитационные взаимодействия все равно слишком разнородные даже в рамках классического поля, чтоб говорит о их объедение.
А условия при которых гравитация и сильное взаимодействия становится одного порядка это уже большие макроскопические тела.
Аноним 10/11/22 Чтв 14:17:31 #368 №567481 
>>567480
Так у других же сил другие законы ослабления, нелинейные вообще.
Что кстати странно, херня про "частицы-переносчики" взаимодействия крайне тупо звучит.
Вот обратноквадратичный как ослабление поля, да, супер, прям как мир устроен. А с некоторыми взаимодействиями прям костыль.
Аноним 10/11/22 Чтв 14:21:02 #369 №567482 
Поясните простым языком, что доказал Перельман?
Аноним 10/11/22 Чтв 14:57:43 #370 №567484 
>>567482
Берем 4-мерное пространство и в нем строим сферу по тем же правилам как мы строим сферу в 3-мерном пространстве. Получаем трёхмерную сферу. Трехмерной ее называют потому, что она локально параметризуются тремя числа. Сфера в трехмерном пространстве называют двухмерной сферой, потому она она образует поверхность - параметризуется двумя числами.

Суть гипотезы в том, что 3-мерную сферу можно плавно и без разрывов деформировать в 3-ых мерный шар у которого исключен край. (Это как с выколотой точкой у отрезка, только тут поверхность вычтена)
Аноним 10/11/22 Чтв 15:13:55 #371 №567486 
>>567481
Большинство сил производные от фундаментальных, поэтому и законы нелинейные.
"частицы-переносчики" это вариант объяснение сложных сил фундаментальными.

И гравитация не является линейной теорей. Только в ньютоновском случае она ослабляется с квадратом расстояния. В очень сильных полях она уже носит экспоненциальный характер. По этой причины коллапсы очень плотных объектом могут остановить квантовое вырождение вещества(поскольку вещество является квантами полей, то при определенных условиях вещество и взаимодействие могут поменяться ролями) или мощные энергетические процессы.
Аноним 10/11/22 Чтв 16:11:15 #372 №567487 
>>567484
Вселенная - 4-х мерная гиперсфера?
Аноним 10/11/22 Чтв 16:41:39 #373 №567489 
>>567468
Ебать откуда ты тут такой прошаренный. Может за LSZ reduction формулу и Ward identity пояснишь на пальцах. Серьезно спрашиваю.
Аноним 10/11/22 Чтв 17:26:57 #374 №567490 
image.png
image.png
image.png
>>566163 (OP)
Тупой вопрос: Что это за прибор?
На корпусе никаких обозначений, кроме знака "Альфа-излучение!" и какого-то логотипа. Корпус пластиковый, прозрачная крышка - пластиковая, внутри никакого газа быть не может, конструкция не задумана быть герметичной. Под крышкой металический круг, покрытый, предположительно, графитом. Над ним контакт. Был найден вместе с советским оборудованием для опытов по физике.
Аноним 10/11/22 Чтв 17:47:31 #375 №567491 
>>567487
Нет. Вселенная псевдоэвклидовое многообразие, а там уже другие топологические законы работают.
Если Вселенную описать через базис на изотропных векторах, то есть ее отображение на комплексную проективную плоскость. У нее уже совсем другие топологические свойства, чем у гиперсферы.
Аноним 10/11/22 Чтв 18:02:58 #376 №567492 
>>567489
Откуда и все - из пизды мамки.
По корреляционную функцию и приемы работы с ней будет тяжело объяснить на пальцах, ибо по большей части она завязана на бескомпромиссную функциональщину.
Аноним 10/11/22 Чтв 21:31:46 #377 №567494 
>>567492
А что скажешь по поводу энергии вакуума?
1) Одни авторы пишут что эффект Каземира является доказательством ее наличия, другие наоборот утверждают что это чисто электромагнитное явление. Твое мнение?
2) Почему нельзя взять нормально упорядоченный гамильтониан и просто постулировать что он такой отбросив одну вторую из-за которой возникаю все бесконечности - ведь при квантовании порядок операторов все равно не возможно определить из классического предела?
Аноним 10/11/22 Чтв 22:56:41 #378 №567495 
>>567494
Вакуум представляется как совокупность полей и при квантовании их энергия всегда будет ненулевой. Это вытекает из сути квантования как дискретности физического действия, в противном случае поля были бы физически ненаблюдаемы.
1. Не сказал бы, что это прям доказательство, эффект Каземира объясняется огромной кучей разных способов. В самой простом варианте это разжиревшие дисперсионные силы. Как чисто электромагнитное явление он не может быть, ибо у него есть аналоги для фононов в твердом теле.
2) Дирак писал об этом. Всякое упорядочивание операторов по сути разложение функционала в ряд, а порядок членов в ряду очень важен, особенно важен нулевой член, ведь для него тоже есть решение. Уберешь его, у тебя вообще ничего не будет сходиться.
Аноним 10/11/22 Чтв 23:48:25 #379 №567496 
Как загуглить органическую молекулу по схеме, названия которой я не знаю? Ну, вот, скажем, вижу я схему DMT — я, конечно, в целом понимаю, что передо мной нарисовано, могу перерисовать, могу сказать, какие в нём атомы, вижу бензоловое кольцо, но, блядь, сказать, что это какой-то там триптамин, да ещё и диметил, я скорее всего не мог даже в школе — не думаю, что у меня когда-то был словарный запас для именования таких рисунков. А какой-нибудь там эпинефрин ((-)-(1R)-(3,4-Дигидроксифенил)-2-метиламиноэтанол), пиздец нахуй, я даже не помню, что вообще такое "фенил" тем более.

Может какой-нибудь поисковик специальный придумали, где можно рисовать молекулы, не зная всей этой абракадабры?
Аноним 11/11/22 Птн 00:25:09 #380 №567497 
>>567495
А можно по-подробнее что там и где Дирак писал?
И еще вопрос - получается ли что в квантовой механике энергия абсолютна т.к. задает скорость вращения фазы пси-функции через уравнение Шредингера или все равно можно как в классической механике менять нулевой уровень и это не особо влияет на итоговый результат?
Аноним 11/11/22 Птн 01:32:09 #381 №567499 
>>567497
В своих лекциях о КТП или монографии о принципах, точно не помню.
Нет, в квантовой теории абсолютно только действие. Энергия является оператором, который связан коммутационным соотношением с оператором временем. Сдвиг по энергии должен менять эволюцию состояний по времени, ведь это по сути смена базиса. Только в стационарном случае или адиабатическом приближение сдвиг по энергии(или времени) это умножение волновой функции на фазовый множитель. В общем случае этот трюк не работает, поэтому в КТП так важно вычислять конденсат для полей.
Аноним 11/11/22 Птн 02:48:07 #382 №567500 
>>567499
> квантования как дискретности физического действия
> абсолютно только действие
Чет впервые слышу про такое.
Аноним 11/11/22 Птн 05:54:12 #383 №567501 
>>566163 (OP)
А что если пространство вывернуто наизнанку? То есть микромир на самом деле является макромиром, и наоборот. Математически возможно вывернуть пространство, и как это сделать?
Аноним 11/11/22 Птн 12:52:08 #384 №567504 
>>567500
Прямо об этом говорят только при геометрическом квантовании, когда нам нужно задать, что действие квантовой системы не может быть сколько угодно малым как в классической физике.

Косвенно о дискретности действия говорят в фейнманоском интеграле по траектории, но там больше акцент на безразмерной комплексной величине - фазе, которая домножается на действие .

И о дискретности/порционности физических величин думал сам Планк, когда пилил модель чернотельного излучения, не зря постоянная его имени имеет размерность действия.
Аноним 11/11/22 Птн 13:41:37 #385 №567505 
>>567504
А можешь прям ткнуть куда нибудь чтобы было прям прямым текстом написано про абсолютность действия, его квантование и
>действие квантовой системы не может быть сколько угодно малым как в классической физике
Чет у меня есть подозрения что ты какую то дичь втираешь.
normal ordering Аноним 11/11/22 Птн 16:50:12 #386 №567506 
Screenshot-642.png
Screenshot-643.png
Мне кажется или тут кто то трындит.
Аноним 11/11/22 Птн 18:23:07 #387 №567509 
Почему до сих пор не построили квантовую теорию гравитации? Чего не хватает? Когда построят?
Аноним 11/11/22 Птн 18:45:08 #388 №567511 
274px-Close-upofSirius.jpg
Не могу найти в гугле ответ на этот вопрос.

Может ли звезда изменить цвет наблюдаемый с земли на другой, если изменится её направление или скорость? Например сириус движется в сторону солнца и мы видим его сейчас бело-синим, а если развернуть его направление на 180 градусов, то может ли он стать красным или желтым?
Аноним 11/11/22 Птн 19:30:03 #389 №567512 
>>567509
Главная проблема гравитация это не поле. И если даже свести ее к полю, то оно не квантуется привычным способами как другие поля.
Тут нужны другие подходы к квантованию вроде переформулировки всей квантовой теории(типо струн) или мы используем другую геометрию для гравитации(вроде петлей). Не хватает экспериментального подтверждения, чтоб узнать какой подход верен и в каком направление дальше развиваться.
Аноним 11/11/22 Птн 20:00:10 #390 №567513 
>>567511
Да может, но эта звезда должна двигаться с релятивистской скоростью от нас.
Аноним 11/11/22 Птн 22:57:37 #391 №567516 
Сап.
Вот я тут подумал, что если существует такое состояние матери, когда те базовые частицы, из которых строится ядро атома, являются химическими элементами нулевой группы? Возможно ли это при отсутствии, допустим, гравитации как таковой?
Аноним 11/11/22 Птн 22:58:12 #392 №567517 
>>567516
Материи.
Аноним 11/11/22 Птн 23:45:21 #393 №567518 
>>567504
> модель чернотельного излучения
Кстати у Шварца в самом начале книги есть "вывод" этого излучения. Только он предусмотрительно сразу отбрасывает 1/2 и вообще о них не заикается, иначе там все по пизде пойдет.
Аноним 12/11/22 Суб 00:02:04 #394 №567519 
>>567516
Что за бредятина? Это нейросеть генерит вопросы из случайных слов?
Аноним 12/11/22 Суб 00:06:24 #395 №567520 
>>567519
Нет.
Вот представь, кварки без гравитации образуют иные атомы, допустим, с массой в 1/2 от массы протона. И так далее. И все это СТАБИЛЬНО. Только условия должны подойти. Если полностью убрать гравитацию, в т.ч. - самих частиц, возможен ли такой эффект?
Аноним 12/11/22 Суб 00:11:18 #396 №567521 
Почему если сложить синий и красный, получится фиолетовый?
Аноним 12/11/22 Суб 01:15:49 #397 №567522 
>>567520
Как гравитация влияет на кварки, блядь?
Ты различаешь четыре фундаментальных взаимодействия или двачерам, чтобы высраться, это необязательно?
Гравитация притягивает Луну к Земле. Кварки, конечно, тоже гравитируют, но настолько слабо, что никакими способами это обнаружить нельзя. Они друг с другом связываются, образуя в т. ч. нуклоны, за счёт сильных взаимодействий.
Ну а химические свойства определяются электромагнетизмом.
Аноним 12/11/22 Суб 01:18:26 #398 №567523 
>>567521
>Почему если сложить синий и красный, получится фиолетовый
Настоящий фиолетовый не получится. Получится что-то похожее, да и то только для человеческого глаза. Пчела, например, ни за что фиолетовый не перепутает со смесью красного с синим.
Аноним 12/11/22 Суб 01:43:17 #399 №567524 
>>567522
Нет, я сказал именно то, что хотел сказать.
Наша вселенная - черная дыра первого уровня, а снаружи элементы состоят из более простых частиц, а какое-нибудь железо там - исключительно радиоактивно и нестабильно.
Аноним 12/11/22 Суб 01:52:43 #400 №567525 
>>567523
Значит ли это что, допустим, кошки видят RGB экраны в искаженных цветах? То есть RGB-оранжевый апельсин и апельсин ирл разных цветов, с их точки зрения?
Аноним 12/11/22 Суб 02:04:34 #401 №567526 
https://dzen.ru/media/id/6367b53e0a83ad7a74b49d4f/o-cifrovoi-degradacii-xxi-veka-6367b5dd44dbb37ff97aa71b
Vajno  [mailto:1] 1 12/11/22 Суб 02:05:32 #402 №567527 
https://dzen.ru/media/id/6367b53e0a83ad7a74b49d4f/o-cifrovoi-degradacii-xxi-veka-6367b5dd44dbb37ff97aa71b
Аноним 12/11/22 Суб 02:26:31 #403 №567528 
>>567501
бамп вопросу
Аноним 12/11/22 Суб 12:09:44 #404 №567533 
Поясните за число Грэма.

Вот 3 = 3^27= 7 625 597 484 987. Тогда пентация 3↑↑↑3 это 3 где количество троек равно как раз = 7 625 597 484 987? Так?
Аноним 12/11/22 Суб 12:17:22 #405 №567534 
>>567533
> Тогда пентация 3↑↑↑3 это

степенная башня из троек, 7 625 597 484 987 цифр в высоту.

Also математические вопросы лучше задавать в /math/.
Аноним 12/11/22 Суб 12:40:29 #406 №567535 
>>567534
Там мертвая доска
Аноним 12/11/22 Суб 14:21:04 #407 №567539 
>>567534
А гексация тогда что? 3↑↑↑↑3 = число троек в башне, посчитанное после 3↑↑↑3?
Аноним 12/11/22 Суб 14:34:31 #408 №567540 
>>567539
3↑↑↑↑3 = (3↑↑↑(3↑↑↑3))
Аноним 12/11/22 Суб 15:05:13 #409 №567541 
Screenshot3.png
Как называеться такой тип одежды?
Аноним 12/11/22 Суб 15:18:26 #410 №567542 
Безіменний.png
Мне нужны текст фразо-шаблоны простых и сложных оправданий и забытых приемов доказывания какой либо точки зрения.
Ну хотя бы класические.
У меня просто настолько ужастная апатия что я даже не могу думать полностью именно нормально в эти дни.
Аноним 12/11/22 Суб 16:12:06 #411 №567544 
>>567541
Пальто или плащ.
Аноним 12/11/22 Суб 16:15:24 #412 №567545 
>>567542
>простых и сложных оправданий
И ЧЁ?
Универсальное, об него разбиваются любые аргументы.
Аноним 12/11/22 Суб 16:26:48 #413 №567546 
>>567542
За достоинства уважают, а любят за недостатки.
Аноним 12/11/22 Суб 16:57:40 #414 №567548 
9g4XEw7xbOM.jpg
>>567546
Тонко.
>>567544
Ты будешь очень удивлен если я тебе скажу что за целую свою 30 летнюю жизнь почти не больше 2х раз в школе не видел ни одного человека в такой одежде, ни в достаточно так городской школе ни во время учебы в университете, ни за 8 лет рабочей жизни, ни на улице, ни на дороге, нигде, только в училок несколько раз, всего , уроды, ржака, пиздец..
Аноним 12/11/22 Суб 18:14:35 #415 №567549 
>>567542
ТАК ПОЛУЧИЛОСЬ
100 процентный детектор пиздлявого пидораса
Аноним 13/11/22 Вск 04:51:13 #416 №567560 
>>567548
Они попадаются довольно редко, вышли из моды к сожалению. А ведь так хочется иногда надеть плащ, федору и взять в руки томми-ган...
Аноним 13/11/22 Вск 05:48:52 #417 №567561 
download (1).jpg
1. Имеют ли магнитотерапия и электрофорез доказанный лечебный эффект?
2. Какие диагнозы, практики и препараты постсоветской медицины - очевидная шарлатанская чепуха? За исключением очевидных "фуфломицинов".
3. Почему они появились в советской медицине, если к тому не было экономических стимулов?
4. Почему хотя бы от некоторых из них не избавились в приказном порядке? Сомневаюсь, что у магнитотерапии есть коммерческое лобби. В чём дело?
Аноним 13/11/22 Вск 06:10:37 #418 №567562 
>>567561
>если к тому не было экономических стимулов?
Стимулы то как раз могли быть чисто экономические: заинтересованный товарищ проталкивает свою разработку потому что это дешево и можно легко вставить в какой-нибудь пятилетний план.
Аноним 13/11/22 Вск 07:07:25 #419 №567563 
>>567562
Без примеров это не объяснение.
Аноним 13/11/22 Вск 07:23:00 #420 №567564 
Принцип запрета Паули говорит в частности, что фермионам запрещено быть как бы в одном месте? А так как все что мы встречаем каждый день это фермионы, то мир кажется твердым? А типо бозоны могут летать как хотят, но просто обычно вступают в реакцию с веществом? Но вот например я слышал , что нейтрино летает сквозь планету туда сюда, но оно фермион. Получается моя теория не верна?
Аноним 13/11/22 Вск 08:11:51 #421 №567566 
посоны рили тупой вопрос как назвать переменную в процессе, чтоб подчеркнуть, что она является не только частичной причиной, но и следствием других процессов? Например, нейропластичность, благодаря которой ты не только закрепляешь целевые навыки, но которая влияет на поведение гораздо шире и делает тебя более склонным, мотивированным эти навыки развивать дальше (или даже забить на них, но закрепить упорство как таковое). Динамическая переменная или как?
Аноним 13/11/22 Вск 08:53:57 #422 №567567 
>>567564
>как бы
Если в твоей "теории" есть эти два слова, то обсуждать ее будешь сам с собой.
Эти слова употребляются, когда человек сам не знает, что хочет сказать.
Возьми общепринятое определения статистики Ферми-Дирака и попробуй ещё раз сделать выводы.
Аноним 13/11/22 Вск 13:13:01 #423 №567572 
>>567561
Недавно осознал что ПОЛОСКАНИЕ ГОРЛА это чисто совковая мулька и никто в мире этим не занимается. Ведь стоит только задуматься на секунду и понятно что это просто какая то ебанутость. Пытался донести эту идею окружающим и конечно же на меня смотрели как на ебнутого - ну как же вот диды полоскали и все врачи всегда пишут полоскать, а ты не умничай.
Аноним 13/11/22 Вск 14:27:50 #424 №567573 
download.png
>>567572
Японцы занимаются. Съел, контра?
Аноним 13/11/22 Вск 14:56:11 #425 №567574 
>>567572
Потому что в совке не было рынка и бизнеса и врачи за откаты не раздували рецепты. Тогда выписывали только необходимое, например, парацетамол и полоскание. А теперь нахуярят гомеопатии, дорогих аналогов и спреев для горла. Т.е. из 50 рублей на обычный парацетамол и бесплатного полоскания раздувают бюджет лечения на косарь в легкую. При этом спреи для горла только удобны тем, что можно в любом месте побрызгать, но эффективность по сравнению с полосканием слабенькая.
Аноним 13/11/22 Вск 15:01:09 #426 №567575 
>>567573
Ты сам откуда знаешь это, из японии капчуешь?
Натыкался на такую инфу, но только они этой хуйней регулярно призывают заниматься а не во время болезни - разные сорта шизы. И даже какое то "исследование" есть что если по десять раз в день гарглить (подчеркиваю - каждый день, а не только когда заболел) то вероятность заболеть снижается на какой то там процент. Предполагаю что происходит это в основном от того что чтобы по десять раз гарглить не получится вообще от раковины отходить далеко.
Аноним 13/11/22 Вск 15:02:54 #427 №567576 
>>567574
Ебать ты дебс промытый, на швятой совочек дрочишь.
Аноним 13/11/22 Вск 15:42:16 #428 №567577 
FSxxtISML0w.jpg
>>567549
Именно. Но я к тому же еще и слегка даун.
>>567560
Я уеду жить в Лондон, или как там.
Аноним 14/11/22 Пнд 03:13:28 #429 №567596 
Объясните прикладное значение гипотезы Римана, т.е. что именно даст ее доказательство. Допустим, мы сможем уверенно определять точное количество простых чисел <= любого N… Так, и что с этого?
Аноним 14/11/22 Пнд 05:04:02 #430 №567597 
>>567563
Партия приказывает тебе найти лекарство от болезни x. А ты не можешь, ресурсов не хватает. Твои действия.
Аноним 14/11/22 Пнд 06:30:30 #431 №567598 
>>567597
Каких ресурсов не хватает? В послевоенном СССР была гигантская сверх-централизованная сеть из санэпидемстанций, поликлиник, больниц, специализированных клиник, НИИ, плюс чудовищных размеров армия с собственной медициной и наукой.
Другое дело, что врачи были низкоквалифицированными, а качество мед. услуг - плохое. Но низовое звено врачей и чиновников не определяет, какими методами пользоваться, система централизована. Мед. наука была вполне на уровне, уж могли отличить работающие процедуры от неработающих.
Аноним 14/11/22 Пнд 06:30:37 #432 №567599 
>>567597
Каких ресурсов не хватает? В послевоенном СССР была гигантская сверх-централизованная сеть из санэпидемстанций, поликлиник, больниц, специализированных клиник, НИИ, плюс чудовищных размеров армия с собственной медициной и наукой.
Другое дело, что врачи были низкоквалифицированными, а качество мед. услуг - плохое. Но низовое звено врачей и чиновников не определяет, какими методами пользоваться, система централизована. Мед. наука была вполне на уровне, уж могли отличить работающие процедуры от неработающих.
Аноним 14/11/22 Пнд 08:06:34 #433 №567600 
>>567599
>Каких ресурсов не хватает?
Мозгов, человекочасов, нужных специалистов, оборудования и расходных материалов, удачи.
Какая разница, исследование и разработка вообще плохо поддаются планированию. Но план-то есть, неудача требует козла отпущения, а составители сидят повыше исполнителей. Так что или гони лекарство, или прощайся с карьерой в лучшем случае.
Аноним 14/11/22 Пнд 09:54:57 #434 №567608 
>>567600
>Мозгов, человекочасов, нужных специалистов, оборудования и расходных материалов
Всего этого в советских НИИ было до пизды.
>план-то есть
План на изобретение новых методов лечения? Таких планов не было.
Кроме того, физиотерапию или магнитотерапию изобрели не в СССР. И это дорогостоящие для массового внедрения процедуры.
Аноним 14/11/22 Пнд 13:15:48 #435 №567632 
>>567505
>>567506
Почему никто не отвечает? Сложновато? Куда то мгновенно сливаются все доктора по квантовым оперденям стоит только спросить что то посерьезнее чем как ежики ебутся. Но зато тут же за квантовую гравитацию попиздеть это мы можем.
Аноним 14/11/22 Пнд 14:05:02 #436 №567634 
>>567632
А что там не понятого. Типичная вольность физиков.
Нормальное упорядочивание это сродни экспоненцированию, в общем случае он неверен, но поскольку мы специально не берем сложные случаи, то и по дефолту считаем все ок.
Аноним 14/11/22 Пнд 14:33:14 #437 №567635 
1.mp4
2.mp4
c — скорость света
Z — количество протонов в ядре
1. "скорость" ближайшего к ядру электрона в водороде == c/137?
2. "скорость" ближайшего к ядру электрона в ядре с Z=137 по известным правилам должна быть == c?
3. "скорость" ближайшего к ядру электрона = c/Z?
Аноним 14/11/22 Пнд 14:38:20 #438 №567636 
>>567635
3. Z/137?
Аноним 14/11/22 Пнд 15:16:50 #439 №567637 
>>567634
Как понять перед нами случай простой или сложный?
Аноним 14/11/22 Пнд 15:39:59 #440 №567639 
>>567637
Смотреть расходится ряд или нет.
Но обычно используют все наоборот, строят заведомо сходимый ряд, члены у которого имеет строгие соотношения между собой. Так получаем заведомо нужные случаи.

Ну или используют методы представлений групп Ли, беря все готовое.
Аноним 14/11/22 Пнд 15:50:29 #441 №567641 
>>567639
Ясно, короче написал бы лучше просто что нихуя не знаешь как и предыдущий клоун, тут все свои.
Аноним 15/11/22 Втр 00:10:53 #442 №567653 
Так я, бля, не понял. Электроны движутся по орбитам или нет?!
А то как читаешь или смотришь какое-нибудь видео, люди всегда поправляются, мол на самом деле электроны там не движутся.

А как же в принципе существование электромагнетизма в системе электрон-ядро? Оно ведь невозможно без СТО и без относительного движения.
Или я вот точно помню, что на твердотельной электронике в институте, когда я был студентом, мы облучали кристаллы рентгеном, а потом что-то считали, и там точно был некий поправочный релятивистский коэффициент для электронов прямиком из СТО.
Так движутся или нет?
сч
Аноним 15/11/22 Втр 00:11:38 #443 №567655 
>>567635
137 - прямо магическое число для физиков.
Аноним 15/11/22 Втр 05:50:19 #444 №567659 
>>566215
О! Вот ты то мне и нужен.
Тоже интересует это вопрос. Но сам я очкую за своё здоровье. Поэтому давай ты проведёшь этот эксперимент и потом напишешь итог в этом треде.
Жду и рассчитываю на тебя.
Аноним 15/11/22 Втр 06:02:52 #445 №567660 
image.png
>>566224
Австралопитек больше похож на шимпанзе чем на человека. Так что наврятли.
По моему диванному мнению, общее потомство могло бы быть у сапиенса с эректусом. Это разница в 1.6 миллиона лет. Тащем-то эректусы принципиально от человека отличаются лишь размером мозга (у них как у 6-летних детей) и формой ебальника. Ниже головы мы не отличаемся. Ну и технически мы скорее всего даже не разные виды, а подвиды одного вида, жившие в разное время. С учётом того, что у сапиенсов было общее потомство с неандертальцами и денисовцами и все эти люди возникли из эректусов, то выходит, что сапиенсы, неандертальцы и денисовцы это расы одного вида. Тем паче что и жили одновременно, и ареалы пересекались - это как раз подпадает под определение именно расового разнообразия и отменяет деление на виды и подвиды. Все эти люди - это один вид с - эректусы.
И так получаем подвид homo erectus sapiens (неандертальцы, гомосапиенсы и денисовцы, вместе взятые) и базовый древний подвид homo erectus.
Кто так не считает, тот ретроград и не может в критическое мышление.
Аноним 15/11/22 Втр 06:05:43 #446 №567661 
>>566226
Потому что пиздюлей мало получали. Надо было пиздить с детства, тогда бы выросли нормальные птицы, а не эта вот пидарасня.
Аноним 15/11/22 Втр 06:06:32 #447 №567662 
>>566274
Кёртис долбоёб.
Аноним 15/11/22 Втр 06:10:01 #448 №567663 
>>566282
Нормальность и сумасшествие это относительные и очень субъективные характеристики. Я психоэмоционально отличаюсь от основной массы 99% людей, являющихся нормальным быдлом. Технически меня можно было бы объявить сумасшедшим. Но это некому сделать, потому что у 99% быдла не хватает мозгов для этого, а оставшиеся 1% такие же сумасшедшие как и я.
Аноним 15/11/22 Втр 06:10:50 #449 №567664 
>>566308
В разделе /hi/ долгий тред на эту тему, уже +100500 перекатов.
Аноним 15/11/22 Втр 06:13:49 #450 №567665 
>>566358
Ну во первых нет солнца = нет фотосинтеза = нет кислорода.
Так что никаких мхов. Только бактериальные маты.
Аноним 15/11/22 Втр 06:15:54 #451 №567666 
>>566370
Должна быть прочнее чем у слона. Слона можно заебашить из крупнокалиберки.
Аноним 15/11/22 Втр 06:16:46 #452 №567667 
>>566412
А сколько ты потребляешь сала в граммах в сутки? Начнём с этого.
Аноним 15/11/22 Втр 06:18:18 #453 №567668 
>>566444
>Как социологии доказывают валидность соц. опросов?
Массово психически воздействуют на людей, заставляя их верить. Говоря по простому - зомбируют. Только так и никак иначе.
Аноним 15/11/22 Втр 06:18:58 #454 №567669 
>>566455
Засекретили.
Аноним 15/11/22 Втр 06:22:14 #455 №567670 
>>566513
Потому что те, кто не любили, оставили мало здорового потомства, которое не выдержало конкуренцию и выродилось.
Аноним 15/11/22 Втр 06:23:15 #456 №567671 
>>566514
Антидепресант превращает тебя в быдло. Так это работает.
Аноним 15/11/22 Втр 06:24:53 #457 №567672 
>>566560
Потому что летит не той стороной. Надо передом, а он брюхом. Конструкция самолёта не приспособлена для управления при полёте брюхом.
Аноним 15/11/22 Втр 06:25:26 #458 №567673 
>>566585
>Почему науку считали и считают хуетой?
Кто? Начнём с этого.
Аноним 15/11/22 Втр 06:28:47 #459 №567674 
>>566601
>= самое важное
Ты так решил?
А кто-то решил, что ему важнее что-то другое. Кто из вас прав? Выяснить это можно только в смертельном поединке. Бейся насмерть с чайлдфри, попытайся его (или её) убить. Если получится и останешься жив - тогда ты прав. Иначе - ты не прав и продолжение рода не самое важное.
Аноним 15/11/22 Втр 06:30:52 #460 №567675 
>>566607
>Провожу мысленные испытания
>Эта херня никуда не едет. Что я делаю не так?
Не так мыслишь. Очевидно же. Сходи к психологу.
Аноним 15/11/22 Втр 06:32:36 #461 №567676 
>>566643
Никуда ток не течёт. Ты нарисовал короткое замыкание.
Аноним 15/11/22 Втр 06:34:28 #462 №567677 
>>566659
>Как дэгэнэраты становятся успешноблядями?
Случайно. Один на миллион.
Аноним 15/11/22 Втр 06:35:55 #463 №567678 
>>566666
Потому что магнит тянется назад с такой же силой, с какой машина тянется вперёд. Силы уравновешиваются.
Аноним 15/11/22 Втр 06:39:07 #464 №567679 
>>566725
В твоей задаче многое не понятно. "проводник" закрытый или есть ещё дырочка, куда можно вытекать?
Кроме раствора в "проводнике" ещё что-нибудь есть кроме раствора, например воздух (или вакуум)?
Аноним 15/11/22 Втр 06:39:44 #465 №567680 
>>567653
Дай определение движения, и я тебе отвечу
Аноним 15/11/22 Втр 06:41:12 #466 №567681 
>>566732
Гугли две вещи.
1. Какая минимальная концентрация урана должна быть, чтобы топливо считалось пригодным.
2. Какая нормальная заводская концентрация урана в урановом топливе.
3. Период полураспада урана.
Дальше берёшь ексельку и считаешь.
Аноним 15/11/22 Втр 06:45:01 #467 №567682 
>>566801
Потому что большинстве преступлений у них случаются в гетто. Гетто это специально выделенная территория для долбоёбов. Так надо. И нехуй на них тратить полицейские ресурсы. Исключи из статистики гетто и тогда раскрываемость и наказуемость окажутся заметно больше.
Аноним 15/11/22 Втр 06:47:02 #468 №567683 
>>566908
Здание не герметичное. Воздух протекает внутрь и продолжает давить.
Аноним 15/11/22 Втр 06:48:32 #469 №567684 
>>566916
Еда была такая, что были нужны такие хуёвины. Примерно так.
Аноним 15/11/22 Втр 06:49:22 #470 №567685 
>>567682
Чел, гетто - район с высокой преступностью, больше ничего. При этом 99% населения гетто - это, например, не убийцы. Полиция игнорирует убийства, т.е. не делает свою работу, всё более превращая район в гетто. Великолепная защита полиции.
>Исключи из статистики гетто и тогда раскрываемость и наказуемость окажутся заметно больше.
Типичная полицейская логика.
Аноним 15/11/22 Втр 06:49:32 #471 №567686 
>>566941
Может читать газетный шрифт с дистанции 1 метр.
Аноним 15/11/22 Втр 06:50:51 #472 №567687 
>>566941
Моя тян с очень хуёвым зрением видела, например, здания, расположенные на расстоянии несколько километров. Хуёво, но видела.
Аноним 15/11/22 Втр 06:52:02 #473 №567688 
>>566954
Попробуй в форме хуя https://ru.wikipedia.org/wiki/Бионика
Аноним 15/11/22 Втр 06:55:20 #474 №567689 
>>567100
>рука чувствует резкий удар.
>Но это ведь газ, он должен идеально равномерно и гладенько сжиматься, с постепенным наростонием усилия.
Просто ты тормоз по жизни и не успеваешь заметить идеально равномерное и гладенькое сжатие, т.к. онно происходит быстрее чем ты успеваешь заметить.
На самом деле нет. При ударе об стол в воздухе возникает ударная волна. Вот её и чувствуешь
Аноним 15/11/22 Втр 06:55:58 #475 №567690 
>>567149
В них газики накапливаются.
Аноним 15/11/22 Втр 06:56:29 #476 №567691 
>>567152
А в макарошках газики не накапливаются.
Аноним 15/11/22 Втр 06:57:10 #477 №567692 
>>567164
Плати 1000 р. Тогда решу.
Аноним 15/11/22 Втр 07:01:43 #478 №567693 
>>567253
Нужно очень солёно поссать. Обычные человеческие ссаки не достаточно солёные и улитка успевает съебаться до получения критического урона.
Вот если посадить улитку в бутылку, нассать туда, закупорить и сесть на неё, тогда улитка сдохнет.
Или можно попытаться повысить солённость своих ссак. Но тогда скорее всего ты сам сдохнешь раньше, чем поссышь на улитку.
Аноним 15/11/22 Втр 07:02:41 #479 №567694 
>>567290
Борда +18. Съеби. В 5 классе тебе всё расскажут.
Аноним 15/11/22 Втр 07:05:45 #480 №567695 
>>567337
Одночлен это частный случай многочлена с n=x, где х=1.
Т.е. одночлен это всегда многочлен. Хоть складывай, хоть приводи в какой-то там вид. Так что можно даже не складывать ине приводить, всё равно многочлен получишь. Об этом ты узнаешь, если поступишь в универ на техническую или естественно научную специальность. А если на гуманитарное поступишь, то так и останешься одночленовым невеждой.
Аноним 15/11/22 Втр 07:12:37 #481 №567696 
>>567359
Я пришёл тебе напомнить, что ядерная ракета "Сармат" летит всегда без пилота, т.е. технически является БПЛА. Пусковая установка ракеты "Сармат" технически является пушкой (просто закопана в землю и направлена всегда вверх).

>почему из пушки не запускают бпла
Просто ты тупой.
Аноним 15/11/22 Втр 07:14:50 #482 №567697 
>>567685
>99% населения гетто - это, например, не убийцы
Зато долбоёбы.
>>>Гетто это специально выделенная территория для долбоёбов. Так надо. И нехуй на них тратить полицейские ресурсы.
Аноним 15/11/22 Втр 07:23:17 #483 №567698 
>>567697
>пуксреньк
Это "аргумент" от эмоций, чмоня.
Аноним 15/11/22 Втр 07:40:19 #484 №567699 
>>567698
>"аргумент" от эмоций
>чмоня
Эмоции вместо аргументом, как видно, из нас двоих только у тебя.
А с долбоёбами всё сделано логично и правильно.
Аноним 15/11/22 Втр 07:41:40 #485 №567700 
>>567445
Да.
Аноним 15/11/22 Втр 07:42:54 #486 №567701 
>>567446
>взорвется на хер, а не "раскукожится"
А вчём разница? Похоже что взрыв это просто очень быстрое раскукоживание.
Аноним 15/11/22 Втр 07:45:16 #487 №567702 
>>567455
>переход через состояние нейтронной звезды превращает все элементы в водород?
Нет конечно.
Как они выйдут из нейтронной звезды? Никак нахуй. Её даже чёрной дырой не порвать. Дыра всосёт нейтронку всю целиком сразу.
Аноним 15/11/22 Втр 07:47:06 #488 №567703 
>>567482
Он доказал, что он долбоёб.
Аноним 15/11/22 Втр 07:49:22 #489 №567704 
>>567501
Надо начать с простого. Попробуй вывернуть наизнанку свой анус и посмотри что из этого получится. Сделай выводы. Спроецируй выводы на мир.
Аноним 15/11/22 Втр 07:50:28 #490 №567705 
>>567509
Потому что тупые. Очевидно же.
Кому надо, те давно уже построили. Просто тупым не рассказывают.
Аноним 15/11/22 Втр 07:55:48 #491 №567706 
>>567516
Технически это возможно. Можно делать и таки сделали молекулы из 2/3 частиц вместо атомов. https://ru.wikipedia.org/wiki/Экзотический_атом.
Прост никому это нахуй не нужно, кроме 1.5 шизиков. Так что нехуй засирать таблицу неполноценными элементами.
Аноним 15/11/22 Втр 07:56:16 #492 №567707 
>>567541
Бичовский.
Аноним 15/11/22 Втр 07:57:12 #493 №567708 
>>567549
ТАК СОВПАЛО
Аноним 15/11/22 Втр 08:01:05 #494 №567709 
>>567653
>Электроны движутся по орбитам или нет?
Нет. Они случайно ебланятся туда-сюда. Орбит нет, есть орбитали - 3D области, где случайное ебланство электрона более вероятно.
Аноним 15/11/22 Втр 08:15:29 #495 №567710 
>>567680
Ну это когда объект плавно во времени меняет свои географические координаты в сущем пространстве. "Географические" в широком научном смысле, конечно же, когда даже точки, удалённые друг от друга на расстояние в 1 ангстрем тоже считаются географически разными.
надеюсь давать определение слова "сущее" не надо.
Аноним 15/11/22 Втр 08:37:30 #496 №567719 
>>567653
Электроны занимают сразу всю орбиту, как вращающееся кольцо.
Аноним 15/11/22 Втр 08:39:24 #497 №567720 
>>567719
Это тебе так кажется. А на самом деле нет. Хотя бы потому что орбиты нет.
Аноним 15/11/22 Втр 10:07:48 #498 №567739 
16593972783080.jpg
>>567707
реально очень смешно. если бы я еще только мог понять шутку до конца - то было бы еще больше смешно.
Аноним 15/11/22 Втр 10:23:43 #499 №567744 
>>567739
От слова "бич" - человек с характерным бичовским образом жизни.
Аноним 15/11/22 Втр 12:16:33 #500 №567751 
>>567710
> Ну это когда объект плавно во времени меняет свои географические координаты в сущем пространстве. "Географические" в широком научном смысле, конечно же, когда даже точки, удалённые друг от друга на расстояние в 1 ангстрем тоже считаются географически разными.
Тогда электрон не движется. Ибо плавного изменения координат у него нет.
Аноним 15/11/22 Втр 14:39:10 #501 №567759 
>>567751
Если электрон достаточно часто изменять, то он будет часто локализоваться около некоторого положения. Если в добавок будем измерять скорость, то ее частое значение будет прям скоростью предыдущего положения. В среднем электрон оказывается вполне движется.
Аноним 15/11/22 Втр 14:55:40 #502 №567761 
>>567751
Вот ты серьезно на веруна какого-то похож. Ебучие квантовые веруны, постоянно свои мантры повторяете, а осмыслить происходящее не можете.
Аноним 15/11/22 Втр 15:38:45 #503 №567769 
>>567751
Свободный электрон вполне себе движется. А вот в атоме электрон не движится. Он там ебланит туда сюда рандомно скачками.
Аноним 15/11/22 Втр 15:39:36 #504 №567770 
16593932846030.png
>>567744
да, все очень верно, тепер поясни мне хорошо этот термин в его разновидностях каким он бывает, я его определение не могу налуркать уже годами, наверное 5 лет.
Аноним 15/11/22 Втр 15:42:14 #505 №567771 
>>567770
Применительно к одежде бич одевается во что попало вместо покупки новой модной. Например может напялить на себя старое пальто, купленое 20 лет назад его другом и забытое у него на хате во время пьянки. Как на исходной картинке.
Аноним 15/11/22 Втр 15:50:26 #506 №567773 
>>567769
В атоме электрон тоже вполне классически движется, только с отрицательной энергией. Хоть это бредово, но при расширение координат на комплексные числа такое движение становится возможным. Скорость тогда получается чисто мнимой, да еще неоднозначной.
Аноним 15/11/22 Втр 15:52:02 #507 №567774 
>>567773
>Скорость тогда получается чисто мнимой
Т.е. скорости нет, потому что вместо движения случайное ебланство скачками туда сюда. Что и требовалось доказать.
Аноним 15/11/22 Втр 15:53:40 #508 №567775 
Screenshot2.png
>>567771
Жизненно нахуй.))))
Тоесть это просто странно и немодно одевающаяся особь?
Аноним 15/11/22 Втр 15:55:28 #509 №567776 
>>567775
Вполне нормальных бич. Миллионы их.
Аноним 15/11/22 Втр 15:59:27 #510 №567777 
>>567775
В мое время слово "Бич" было синонимом бомжа.
Аноним 15/11/22 Втр 15:59:28 #511 №567778 
>>567776
Возможно даже двачер.
Аноним 15/11/22 Втр 16:03:29 #512 №567779 
>>567777
В школьные годы штоли.
Эти слова имеют разный смысл. Взрослые люди знают и всегда знали разницу. БОМЖ - сокращение Без Определённого Места Жительства, т.е. бездомный. Возникло в 1990е.
БИЧ - сокращение Бывший Интеллигентный Человек, возникло дав но в СССР какпротивопоставление людям рабочего класса. Вместе с развалом ССР и отменой советских классов слово бич утратило изначальное значение и стало применяться ко всем людям на дне общества, в том числе и к заводобыдлу.
Аноним 15/11/22 Втр 16:09:29 #513 №567780 
>>567774
Почему же нет. Скорость есть и даже конкретная. Из соотношения этой скорости и расстояние от ядра получается конкретное значение углового момента.
И если набрать много наблюдений, то средняя скорость получается вполне себе классической скорость средней координаты. Орбиталь не зря прозвали орбиталью.
Аноним 15/11/22 Втр 16:16:32 #514 №567781 
image.png
image.png
>>567780
>И если набрать много наблюдений
Если набрать много наблюдений, то даже случайно падающие шарики выстраиваются в чоткое гауссово распределение. Но это не значит, что они специально так падают. Они всё равно ебланят в случайные места. И когда электрон проебланит, допустим мильён раз, то получится некоторое 3-мерное распределение вероятности где он может оказаться чаще и где реже. Это и называется орбиталь. Которая, кстати, бывает весьма странной форы, совсем не похожей на орбиту. Но это не значит что электрон движется равномерно. Он ебланит квантовыми скачками по своим потенциальным ямам, как любая нормальная квантовая частица.
Аноним 15/11/22 Втр 16:20:49 #515 №567782 
image.png
>>567781
Блять, не та картинка. Вот та.
Видно, что орбитали не просто какие-то чёткие фигуры, а наоборот, расплывчатые. Что как бы намекает на вероятностное поведение электрона. Т.е. в любой момент времени он может оказаться где угодно совершенно случайно. В отличие от равномерного движения.
Аноним 15/11/22 Втр 21:15:16 #516 №567788 
>>567665
Так я потому и написал в ковычках "мох". Надо укрыть сушу неким живописным ковром, чтобы он закачивал обогащенную кислотную воду вглубь суши и за счет этого процветала разнообразная сухопутная жизнь, занимающаяся потребительством и/или хемосинтезом. Разумеется, это должно быть полезно и морским продуцентам.
Аноним 15/11/22 Втр 21:45:59 #517 №567789 
>>567710
>объект плавно во времени меняет свои географические координаты в сущем пространстве
Электрону в атоме нельзя дать четкие x,y, z поэтому меняй определение.
А еще он не обладает координатами и скоростью одновременно, т.е. понятие траектории тоже не имеет смысла.
Всё что ты можешь узнать, это среднюю скорость (она ноль в атоме), скорость среднеквадратичную (она не ноль), момент по осям (от орбитали будет разный) и т.д. Заметь, я использую слово "среднюю".
Аноним 16/11/22 Срд 00:39:45 #518 №567793 
>>567782
С какой скоростью он там ебланит?
Аноним 16/11/22 Срд 01:38:34 #519 №567798 
>>567793
+100500 проебланиваний в наносекунду.
Аноним 16/11/22 Срд 01:39:24 #520 №567799 
>>567788
Дохуя хочешь.
Аноним 16/11/22 Срд 02:04:25 #521 №567805 
>>567793
С какой частотой испусткает фотон - стакой и ебланит. У разных атомов в разных состояниях по разному.
При комнатной температуре это обычно сотни или тысячи терагерц.
Аноним 16/11/22 Срд 02:12:02 #522 №567806 
>>567799
Ну, почему же. Я использую аналогию тепловой машины, которая работает за счет того, что одна ее часть холодная, а другая - горячая. Только вместо тепловой энергии здесь разность электрохимических потенциалов и концентраций между насыщенной сложной органикой кислотной водой морей и пресными атмосферными осадками. Т.е. "мох" нужен а) для транспорта веществ, б) чтобы помешать смешиванию и нейтрализации двух видов воды (что привело бы фактически к бесполезному рассеянию энергии в пространстве). В результате, энергия тратится на жизнедеятельность сухопутной жизни - и это полезно морской части "планетарной машины", ведь суша снижает кислотность воды и обогащает ее необходимыми морским биомам минеральными веществами. Т.е. обе эти части работают на то, чтобы максимум энергии планетарного ядра буферизировался в виде биомассы. Которая, конечно, в ходе естественно отбора развивается и делает планету куда более интересной для моих космических колонистов.
Аноним 16/11/22 Срд 03:16:40 #523 №567813 
image.png
>>567806
У тебя нет солнца - внешнего источника энергии, облучающего всю поверхность планеты, долбоёб! Какой нахуй мох? Твой максимум это плесень вокруг гейзера.
Аноним 16/11/22 Срд 03:32:33 #524 №567814 
>>567813
> Какой нахуй мох?
Тот мох, что у тебя в голове, очевидно. Не пиши мне больше, пожалуйста.
Аноним 16/11/22 Срд 05:42:19 #525 №567815 
>>567814
Обиделся как баба.
Аноним 16/11/22 Срд 10:40:29 #526 №567823 
На генераторе сигналов есть ручка амплитуды, она никак не пронумерована, только max и min, если её потянуть, то амплитуда сильно упадет, как я понял мы отнимаем 20 Дб , т.к там снизу подписано -20Дб. Как связана амплитуда периодического сигнала и децибелы?
Аноним 16/11/22 Срд 11:10:41 #527 №567826 
Поскольку тред вопросов тупых, то не обессудьте.

Есть мнение, что переход от описания символами - буквами к описанию образами картинками, ведет к деградации интеллекта.
Есть ли какие-нибудь исследования, доказательства или примеры из истории на эту тему?
Аноним 16/11/22 Срд 12:04:55 #528 №567829 
>>567823
>-20Дб
=1/10
>+20Дб
=*10

>>567826
Да, оно убирает понятийную составляющую мышления.
Аноним 16/11/22 Срд 12:28:17 #529 №567831 
>>567829
Так а кто и когда и на ком это изучал?
Аноним 16/11/22 Срд 12:29:09 #530 №567832 
>>567823
Там логарифмическая шкала в основном используется.
Аноним 16/11/22 Срд 15:43:06 #531 №567835 
>>567826
Это подвид гипотезы сепира уорфа.
В самом общем случае нет. На когнитивные способности значимо не влияет ни структура языка и ни его письменность, однако в частом случае есть корреляции.
Аноним 16/11/22 Срд 16:12:36 #532 №567840 
>>567826
>примеры из истории на эту тему?
Вновейшей истории корпорация Гугл дала пориджам эмодзи, в которых слова для выражения эмоций заменены скудным набором смайликов так, что для пориджам при выражении разных оттенков эмоций приходится использовать одни и теже смайлики, выражение эмоций получается примитивным и механистичным. В результате пориджи растут эмоционально не развитыми и становятся двачерами и шлюхами.
Аноним 17/11/22 Чтв 07:53:04 #533 №567854 
>>567221
>Любая волна сама себя генерирует - потенциальная энергия переходит в кинетическую циклически
Но в фотоне магнитное поле возбуждается ПРИ НАРАСТАНИИ электрического, а электрическое возрастает ПРИ ВОЗРОСТАНИИ магнитного.
В один момент времени фотон полностью исчезает, и у него нет никаких предпосылок появиться вновь.
Аноним 17/11/22 Чтв 08:12:55 #534 №567855 
>>567512
>то оно не квантуется привычным способами как другие поля.
Но если взять квантовую частицу, элементарный переносчик, которой принято квантовать данное поле,
то на каком-то расстоянии от неё напряжённость поля будет меньше, на каком-то больше.
Получается напряжённость поля не квантуется(как минимум только может планковскими длинами), но поле мы частицами проквантовали ебать?
Хуйня какая-то.

Хотя мне ещё надо в голове как-то связать то что проквантовали, квант поля, и напряжённость поля, хз как они друг между другом связаны и чем являются.
Как бы напряжённость понятно.. но и частица понятна, нг частица как элементарный квант, учитывая что и напряжённость существует, выглядит как бред при этом утверждение о элементарной квантовости частицы данного поля.
Аноним 17/11/22 Чтв 08:14:33 #535 №567856 
>>567520
>Если полностью убрать гравитацию,
А как ты уберёшь если это всё одно?
Аноним 17/11/22 Чтв 08:16:05 #536 №567857 
>>567525
Почитай про монохромное излучение. Видеть предметы в свете солнца и видеть монозромный свет это разные вещи.
Аноним 17/11/22 Чтв 10:48:08 #537 №567859 
>>567856
С помощью анти гравитации, понятное дело же.
Аноним 17/11/22 Чтв 13:30:13 #538 №567862 
2018-12-2013-48-31.png
>>567855
>Но если взять квантовую частицу, элементарный переносчик, которой принято квантовать данное поле,
>то на каком-то расстоянии от неё напряжённость поля будет меньше, на каком-то больше.
Аноним 17/11/22 Чтв 13:52:38 #539 №567863 
>>567855
Ты точно не нейросеть?
Суть квантования это берем некоторую физическую систему и размазываем ее в линейные квантовые состояния. Далее при измерение мы выбираем нужный оператор, который соответствует наблюдаемой величине, которую измеряем.
Поле это считай как обычная квантования система как кубит, только у него ортогональных состояний не два и даже не бесконечность, а бесконечность бесконечность состояний. И для этих состояний еще требуется подобрать нужные операторы, которые соответствует нужной наблюдаемой. Конечно можно подобрать оператор, который дает напряженность как наблюдаемую, но в лоб с функционалами работать тяжело и почти невозможно, поэтому придумали различные способы упростить это дело вроде упорядочивать по определенной величине поля как произведение элементарных порций, собственно квантами полей. Они являются альтернативным взглядом на поле и прямо не связаны его напряженностью. Кванты полей больше говорят о динамике поле, чем о его напряженности.
Теперь почему нельзя гравитацию квантовать как поле. Состояния, которые должны образовывать суперпозицию, порождают абсурдные величины для наблюдаемых. А если использовать трюки как с другими полями, то лезут никак не устранимые расходимости. Везде будут сраные бесконечности. Гравитону нечем подавлять собственное самодействие.
Аноним 17/11/22 Чтв 15:11:32 #540 №567866 
>>567854
Мда. Вот смотри, сила тока это I=U/R. При сверхпроводимости R=0, значит, сила тока бесконечна! Бесконечная сила пронзающая небеса!
Это примерно уровень твоего рассуждения, лол.
Аноним 17/11/22 Чтв 22:54:45 #541 №567875 
>>567862
О, помню этого смешного дебила постящего эту картинку, много раз его обоссывал.
Аноним 17/11/22 Чтв 22:57:17 #542 №567876 
>>567863
Нет, я про то что проквантовав математически они лезут искать реальную частицу-квант и находят.
Получается они просто ложным названием её называют и частица квант это совершенно не то что и каантование поля?
Аноним 17/11/22 Чтв 23:01:20 #543 №567877 
>>567866
Хуета, показывает только что ты не понимаешь что такое электричество.
А про фотон конкретный пример, у него есть частота, происходит наростание и спад.
Если в обычной волне в веществе всё логично, кинетическая энергия спадает к 0 а потенциальная увеличивается к 1, а затем начинает спадать к 0, в то время как кинетическая увеличивается к 1.
То в случае фотона одновременно две составляющие или увеличиваются или спадают, и в какой-то момент обе сотсавляющие становятся 0, и нет никаких предпосылок для их повторного возникновения опять, как переход кинетической в потенциальную или наоборот в случае волны обычной.
Аноним 17/11/22 Чтв 23:54:16 #544 №567879 
>>567876
Нет.
Квант это "порция". Соответственно квантование это "порционирование" и оно может быть вырожено как частицами, а может и другими различными способами например действием некой силы. Все зависит как мы подходим к процессу.
Аноним 17/11/22 Чтв 23:57:54 #545 №567880 
>>567877
Что у фотона нарастает и спадает?
И как быть с круговой и линейной поляризацией, как там делиться у фотона потенциальная и кинетическая энергия?
Аноним 24/11/22 Чтв 19:41:04 #546 №568271 
>>567879
Нет.
Порциями происходит лишь излучение и поглощение, а распространение волновое.
Аноним 24/11/22 Чтв 19:42:12 #547 №568273 
>>567877
>>567880
>фотон
Их не существует, есть лишь акты излучения и поглощения. А все остальные свойства относятся к группам таких актов.
Аноним 25/11/22 Птн 02:34:17 #548 №568282 
image.png
>>566163 (OP)
Можно ли сделать вечный двигатель, на основе видоса из ОП?
Похоже, что да, если прихуярить туда храповик и собачку. Пикрелейтед. Железный шарик, разгоняясь,
бьётся о эту вертушку с четырьмя лопастями,
заставляет её вращаться,
она вращает вал, и вал поднимает груз.
Храповой механизм вала поддерживате собачка, вращая вал только в одном направлении.
Энергия, затраченная при ударе шара о вертушку с четырьмя лопастями,
приводит к тому, что шар падает вблизи края конуса,
с последующим возвращением этой энергии,
за счет гравитационного притяжения шара к центру конуса.

Поясни, почему это не будет работать?
Аноним 25/11/22 Птн 12:15:31 #549 №568294 
>>568282
>Поясни, почему это не будет работать?
Потому что фиговина на видео имеет привод энергии из вне.
Аноним 26/11/22 Суб 04:52:27 #550 №568320 
image.png
>>568294
Где он? По-моему, срабатывает здесь гравитация, как на пикрил.
https://ru.wikipedia.org/wiki/Гравитационная_энергия
И хотя это выглядит просто как замкнутая колебательная система, которая может работать без нагрузки, но если её сделать пиздатой, то по чуть-чуть, через храповик и собачку, можно было бы таки снимать энергию гравитации, не?
Я ещё видел где-то видос подобной замкнутой системы, основанной на капиллярном эффекте. А вот же он: https://www.youtube.com/watch?v=t0-tKY9i9MM

Хотя да, не исключено, что под дощечкой стоит электромагнит с батарейкой, и магнитный выключатель, рядом, придающий дополнительную скорость, шарику.
Аноним 26/11/22 Суб 08:55:53 #551 №568324 
>>568320
>Где он?
Там где его не видно под этим ракурсом
>как на пикрил
Ты подпись то к пикрилу прочитал?
>можно было бы таки снимать энергию гравитации, не?
К сожалению это так не работает
> А вот же он
Полезная в народном хозяйстве штука, жалко что пиздеж.
Аноним 26/11/22 Суб 17:29:32 #552 №568352 
>>568273
То есть у фотона нет геометрии/протяженности?
Аноним 26/11/22 Суб 19:19:02 #553 №568370 
Почему нельзя сделать воду из водорода?
Почему водород называют водород если он не может родить воду?
Аноним 26/11/22 Суб 23:37:17 #554 №568393 
>>568352
>То есть у фотона нет геометрии/протяженности?
В общепризнанной физике у фотона нет массы, для него не течёт время (с ним не происходят события, он не колеблется), а значит он не обладает размерами.

>>568370
>Почему водород называют водород если он не может родить воду?
Тёти тоже рожают не сами по себе (пока что) , а только после оплодотворения широкодоступной спермы. Также и водород рождает воду в условиях широкодоступного воздуха. И название было дано очень давно.
Аноним 27/11/22 Вск 10:52:03 #555 №568401 
>>568393
> В общепризнанной физике у фотона нет массы, для него не течёт время (с ним не происходят события, он не колеблется), а значит он не обладает размерами.
Как тогда его возможно пронаблюдать, если у него нет массы и протяженности? Не является ли в этом случае фотон не объектом, но свойством объекта, как например цвет или твёрдость являются свойством объекта камень?
Аноним 27/11/22 Вск 23:11:34 #556 №568434 
>>568401
>Как тогда его возможно пронаблюдать, если у него нет массы и протяженности? Не является ли в этом случае фотон не объектом, но свойством объекта, как например цвет или твёрдость являются свойством объекта камень?
А его не наблюдают, есть лишь поглощение с передачей энергии и импульса. А вот группа фотонов уже обладают длиной волны и поляризацией.
Аноним 28/11/22 Пнд 09:18:19 #557 №568450 
>>568434
Какая странная штука. Интересно.
Аноним 07/12/22 Срд 22:52:56 #558 №568896 
Почему людям не приятно всё что связано с кровью, внутренностями и другим. Типо это получается боязнь своего тела, но что не так?
Аноним 08/12/22 Чтв 02:12:11 #559 №568904 
>>568896
ХЗ, я свои ссадины и порезы вообще языком облизываю а собаке не облизываю.
Аноним 08/12/22 Чтв 11:55:33 #560 №568920 
>>568896
Внутренности не зря так называются, поэтому когда они становятся наружностями это вызывает дискомфорт.
Аноним 11/12/22 Вск 22:58:35 #561 №569021 
У меня интересный вопрос по ЭПР встал. А можно ли узнать точные координаты фотона? Скорость-то известна.
Аноним 12/12/22 Пнд 16:29:55 #562 №569074 
>>569021
Вот это интересно. По идее, у тебя возникнут проблемы с определением времени прохождения фотона через детектор, так как фотон, рассеиваясь на детекторе, придает тому неизвестный дополнительный импульс, который сделает отметку времени о детектировании неопределенной. Подробнее гуглили что-то вроде "микроскоп Гейзенберга" или как-то так. Там это показано как невозможность определить одновременно координату и импульс освещенного фотоном электрона, но, по сути, это же делает невозможным точно определить сам фотон.
Аноним 27/01/23 Птн 21:45:40 #563 №571408 
>>569021
>У меня интересный вопрос по ЭПР встал. А можно ли узнать точные координаты фотона? Скорость-то известна.
Нет, ты даже радиус волны с которой он едет узнать не сможешь. Потому что в разных средах скорость разная.
comments powered by Disqus

Отзывы и предложения